Sie sind auf Seite 1von 75

REGULATORY FRAMEWORK FOR BUSINESS TRANSACTIONS R.

LOPEZ

OBLIGATIONS & CONTRACTS

Definition of Obligation

 Obligation is a juridical necessity to give, to do, or not to do.

Essential Requisites

 Subject persons- the active subject (creditor or obligee) and the passive subject (debtor or
obligor)
 Prestation or service constituting the object of the obligation- the thing to be given, to be done,
or not to be done.
 Juridical or legal tie (Vinculum juris) – That which binds the parties together.

Sources of Obligation

 Law- Those provided by a particular law- e.g. Payment of tax under the Tax Code
 Contracts- Those agreed upon by the parties- e.g. Lease Contract
 Quasi-contracts
 Delicts (wrongful acts/omissions punished by law)- Civil obligations to pay damages due to
crime committed to another.
 Quasi-delicts- Civil obligations to pay damages due to negligence causing injury to another.

Quasi-Contracts refer to those juridical relations arising from lawful, voluntary and unilateral acts,
based on the principle that no one shall be unjustly enriched at the expense of another.

 Principal kinds of Quasi Contracts:

 Negotiorum gestio- arises whenever a person voluntarily takes charge of the management
of the business or property of another without any power or authority from the latter (also
called unauthorized management).
 Solutio indebiti- arises whenever a person unduly delivers a thing or pays through mistake
to another who has no right to demand or receive it.

 Obligation of the debtor

 In obligation to give determinate thing


 To deliver the thing he promised.
 Accessory obligations
a. To deliver the accessories/accession
b. To deliver the fruits
c. To take care of the thing he promised with the diligence of a good father of a family
(minimum required by law).

 In obligation to give indeterminate thing


 To deliver the quality and quantity agreed upon (not superior nor inferior)

 In obligation to do
 To perform what was promised

 In obligation not to do
 To abstain from what is prohibited

 Remedies of the creditor in case debtor fails to perform his obligation:

 In obligation to give determinate thing


 Compel specific performances; or

1 - 1
REGULATORY FRAMEWORK FOR BUSINESS TRANSACTIONS R. LOPEZ

 Convert the obligation to its cash equivalent


 Ask for other damages

 In obligation to give indeterminate thing


 Ask a third person to perform at debtor’s expense
 Convert the obligation to its cash equivalent; or
 Ask for other damages

 In obligation to do
 Ask a third person to perform at debtor’s expense; or
 Convert the obligation to its cash equivalent
 Ask for other damages
 What was poorly done may be undone at debtor’s expense

 In obligation not to do
 What was done may be undone at debtor’s expense
 Ask for other damages

 In the absence or insufficiency of above remedies:


 Accion subrogatoria- an action of the creditor against the debtors of his debtor.
 Accion pauliana- an action of the creditor to rescind contracts entered by the debtor to
defraud him.

Sources of Liability for Damages

Liability for damages arises when obligation is not performed due to:
 Fraud
 Negligence
 Delay; or
 Contravention of the tenor of the agreement

Fraud here refers to dolo incidente, or fraud in the performance of the obligation. It is also called
malice or bad faith, or the intentional avoidance of performance of the obligation.

Negligence here refers to culpa contractual, or the failure to observe the degree of care required by
the situation resulting to breach of contract or failure of performance of obligation or contract (as
distinguished from culpa aquiliana where there is negligence but no existing contract between the
parties).

Delay here refers to mora solvendi, or the failure of the debtor to perform the obligation when it falls
due (as distinguished from mora accipendi and delay on the part of the creditor to accept).

 Requisites:
 Must be positive obligation
 Obligation is valid, due and demandable
 Creditor makes a demand
 Debtor fails to perform despite of the demand

Rule: No delay if no demand from the creditor judicial or extra-judicial.

 Exception to the rule “no delay if no demand”


When time is of essence in the obligation
When obligation so provides
When law so provides
When demand would be useless (because debtor cannot perform the obligation clearly)
 Failure however to perform the obligation due to fortuitous events exempts the debtor from
liability

Definition of Fortuitous Event

 Fortuitous event is an event which could not be foreseen or though foreseen but inevitable

1 - 2
REGULATORY FRAMEWORK FOR BUSINESS TRANSACTIONS R. LOPEZ

 Requisites:
 Cause is independent of the will of the debtor
 The event must be unforeseeable or unavoidable
 Occurrence must be such nature as to render it impossible for the debtor to fulfill his
obligation in a normal manner
 Debtor must be free from any participation in the aggravation of the injury resulting to the
creditor.

 General Rule:

No liability if debtor fails to perform due to fortuitous event

 Exceptions:
 When expressly declared by law
 When expressly declared by stipulation or contract
 When the nature of the obligation requires the assumption of risk
 When it proceeds from criminal offense
 When the obligation is to deliver a generic thing

Classification of Obligations
Obligations may be primary or secondary.

 Primary obligations are:


 Pure obligations
 Not subject to any condition or period.

 Conditional Obligations
 Subject to a future and uncertain event or a past event unknown to the parties

 Obligation with a period


 Subject to the arrival of a period

 Alternative Obligation
 All are due but only one must be performed

 Facultative Obligation
 Only one is due but can be substituted

 Joint Obligation
 Obligation is pro-rated among the parties

 Solidary Obligation
 One of the parties can collect or can be compelled to pay the entire obligation

 Divisible obligation
 An obligation capable of partial performance

 Indivisible Obligation
 An obligation incapable of partial performance

 Obligation with a penal clause

 Secondary obligations are:


 Real obligations
 Obligation to give or not to give

 Personal obligations
 Obligation to do or not to do

 Unilateral obligations
 Obligation wherein only one party is liable

1 - 3
REGULATORY FRAMEWORK FOR BUSINESS TRANSACTIONS R. LOPEZ

 Bilateral obligations
 Obligation wherein both parties are liable

 Positive obligations
 Obligation to give or to do

 Negative obligations
 Obligation not to give or not to do (no delay here)

 Accessory obligations
 Obligation which depends on another obligation

 Principal obligations
 Obligation which can stand alone

 Conventional obligations
 Obligation based on the agreement of the parties

 Legal obligations
 Obligation based on law

Effects of Loss, Deterioration and Improvement in Real Obligations

 Loss
 Without debtor’s fault- obligation is extinguished
 With debtor’s fault- debtor pays damages

 Deterioration
 Without debtor’s fault- impairment to be borne by the creditor
 With debtor’s fault- creditor may choose between the rescission of the obligation and its
fulfillment with indemnity for damages in either case

 Improvement
 Without debtor’s effort/money or by nature or time- improvement shall inure to the benefit of
the creditor
 At the debtor’s expense- debtor shall have no other right than that granted to a
usufructuary

General Rule on Period

 General rule
When a period is designated for the performance or fulfillment of an obligation, it is presumed to
have been established for the benefit of both creditor and debtor.

 Exception
When it appears from the tenor of the period or other circumstances that it is established in favor
of one or other

 When court may fix period:


 If the obligation does not fix a period, but from its nature and circumstances it can be
inferred that a period was intended by the parties;
 If the duration of the period depends upon the will of the debtor; and
 If the debtor binds himself when his means permit him to do so.

 When debtor loses right to make use of period:


 When after the obligation has been contracted, he becomes insolvent, unless he gives
guaranties or securities for the debt (the insolvency need not be judicially declared);
 When he does not furnish to the creditor the guaranties or securities he promised;
 When by his own act he has impaired said guaranties or securities after their
establishment, and when through fortuitous event they disappear, unless he gives new
ones equally satisfactory;

1 - 4
REGULATORY FRAMEWORK FOR BUSINESS TRANSACTIONS R. LOPEZ

 When debtor violates any undertaking, in consideration of which the creditor agreed to the
period; or
 When debtor attempts to abscond.
 If the obligation contains an acceleration clause
 Other lawful cases as stipulated by the parties.

Effects of Loss of Object in Alternative Obligation

 If right of choice belongs to debtor:

 If through a fortuitous event


 Debtor cannot be held liable for damages

 If 1 or more but not all of the things are lost


 The creditor cannot hold the debtor liable for damages because the debtor can still
comply with his obligation.

 If right of choice belongs to the creditor:

 If 1 of the things is lost through a fortuitous event, the debtor shall perform the obligation by
delivering that which the creditor should choose from among the remainder, or that which
remains if only 1 subsists
 If the loss of 1 of the things occurs through the fault of the debtor, the creditor may claim
any of those subsisting, or the price of that which, through the fault of the former, has
disappeared with a right of damages
 If all the things are lost through the fault of the debtor, the choice by creditor shall fall upon
the price of any 1 of them, also with indemnity for damages.

 Note: If problem is silent:

 Debtor’s choice;
 If debtor cannot choose because of creditor’s fault, he can rescind the contract plus
damages.

Rule on Joint and Solidary Obligations

 Obligation is presumed joint if there is concurrence of two or more debtors and/or creditors.

 Exceptions:
 When expressly stated to be solidary
 When the law requires solidarity
 When the nature of the obligation requires solidarity

Basic Distinctions Between Indivisibility & Solidarity

INDIVISIBILITY SOLIDARITY
 Refers to the prestation or the  Refers to the legal tie or
object of the obligation vinculum juris & the subjects or
parties of the obligation
 Plurality of subjects is not  Plurality of subjects is necessary
required
INDIVISIBILITY SOLIDARITY
 In case of breach, obligation is  When there is liability on the part
converted into 1 of indemnity of the debtors because of the
for damages, indivisibility of the breach, the solidarity among the
obligation is extinguished. debtors remains

1 - 5
REGULATORY FRAMEWORK FOR BUSINESS TRANSACTIONS R. LOPEZ

Important Rules on Divisible & Indivisible Obligations

 Divisibility or Indivisibility of obligation refers to the performance not to the nature.


 The creditor cannot be compelled partially to receive the prestation in which the obligation
consists; neither may the debtor be required to make partial payments.
 Exceptions
 When the obligation expressly stipulates the contrary
 When the prestations constituting the objects of the obligation are subject to different terms
and conditions
 When the obligation is in part liquidated and in part unliquidated

Obligations with a Penal Clause

 Purpose of Penalty
 To ensure the performance of the obligation
 As a form of liquidated damages
 To punish the debtor for the breach of the obligation

 No more damages can be asked if obligation contains penalty except:


 There is a stipulation to the contrary
 Debtor refuses to pay penalty
 Debtor is guilty of fraud

Special Forms of Payment

No. of Parties No. of debts Solvency of


debtor
Application of Payment 1 debtor: 1 creditor 2 or more Solvent
Dacion in Payment 1 debtor: 1 creditor 1 or more Solvent
1 debtor: 2 or more
Payment of Cession creditors 2 or more Insolvent
Tender of Payment &
Consignation 1 debtor: 1 creditor 1 or more Solvent

 In dacion, the delivery of the debtor’s property to the creditor is full payment of the obligation
upon creditor’s acceptance.

 In cession, the delivery is a mere assignment of the debtor to his creditors for them to sell and
to extinguish the obligation but only after proper accounting.

 Consignation shall produce effects of payment only if there is a valid tender of payment.

 Exceptions:
 Creditor incapacitated to receive payment
 Creditor is absent or unknown
 When two or more persons claim the right to collect
 When without just cause creditor refuses to give a receipt
 When the title of the obligation has been lost
 Creditor cannot be located despite reasonable diligence.

Extinguishing Obligations

Obligations are extinguished by:


 Payment or performance
 Must be complete and unconditional to extinguish the obligation

 Loss of the thing due


 If without the debtor’s fault otherwise he is liable for damages

 Condonation or remission of debt


 Creditor gratuitously forgives the obligation of the debtor

1 - 6
REGULATORY FRAMEWORK FOR BUSINESS TRANSACTIONS R. LOPEZ

 Requires debtor’s acceptance to be valid

 Compensation or set-off (Simplified Payment)


 2 persons became principal debtors and creditors of each other
 Does not require agreement of the parties if: (Legal Compensation)
 Both debts are due and demandable
 Both debts are of the same kind and quantity or amount
 Can be availed of by the guarantor even if he is not one of the principal parties.

 Confusion or merger of rights


 A person became the principal debtor and creditor of himself
 Cannot occur if the party involved is the guarantor

 Novation- change in any elements of the obligation


 Real novation- if it is a change in the object or terms of the obligation

 Personal novation
 Expromision- change of the debtor without the old debtor’s consent
 Delegation- change of the debtor with the old debtor’s consent or intervention
 Subrogation- change of the creditor by operation of law
 Conventional novation- change of the creditor by agreement of the parties

Other analogous modes of extinguishing an obligation includes Compromise, Annulment, Mutual


desistance, Prescription, Marriage, Fulfillment of Resolutory condition, Illegality, Impossibility, Death,
etc.

Definition of Contract

 A contract is a meeting of minds between 2 persons for the purpose of creating a juridical
relation.

Essential Elements of Contracts are:

 Consent- the meeting of the minds of the parties


 Object- the thing to be done; not to be done; or to be delivered
 Cause or Consideration- the reason why the parties entered into a contract
Basic Principles Governing Contracts

 Autonomy of contracts
 It means that the parties are free to stipulate anything they deem convenient provided that
they are not contrary to law, morals, good customs, public order and public policy.

 Mutuality of contracts
 It means that the contract must bind both parties; its validity or compliance must not be left
to the will of one of them.

 Consensuality of contracts
 Contracts are perfected by meeting of minds of the parties.

 Relativity of Contracts
 It means that contract will bind only the parties, their assigns or heirs, except in case of
stipulation pour autrui, contracts in favor of 3rd persons wherein the third person must
accept the said agreements in his favor.

 Obligatoriness of contracts
 It means that the contract validly entered upon by the parties has the force & effect of law
between them and must be complied in good faith.

Stages in the Life of a Contract

 Negotiation and conception or generation

1 - 7
REGULATORY FRAMEWORK FOR BUSINESS TRANSACTIONS R. LOPEZ

 Perfection or meeting of minds stage


 Termination or performance by the parties stage (also the extinguishment or death of the
contract

Cognition Theory governs the perfection stage wherein the offeror must learn of the acceptance by
the offeree first.

Factors that Affect Consent and may Result to a Defective Contract

 Incapacity of the parties by nature:


 Ex: Minority, Deaf or mute who cannot read nor write, Insanity or mental incapacity
 Capacity is limited by law:
 Ex: Civil interdiction, unauthorized agents, unreleased insolvents and those prohibited by
law
 With capacity but consent is vitiated by:
 Fraud- refers to dolo consante; fraud in the celebration of contracts
 Mistake- refers to mistake of fact or object (not law) of the contract
 Undue Influence- when one takes improper advantage of his relationship with another
 Intimidation- when one is compelled to act even against his will for fear of a grave evil that
may happen to him, his family or property
 Violence- when one is compelled to act against his will because of force or pain inflicted
upon him.

The Object of a Contract must be:


 Within the commerce of man
 Licit or not contrary to law, morals, good customs, public order or public policy
 Real or with potential existence
 Determinate as to its kind
 Not an impossible service

 Cause of Consideration refers to the extrinsic reason why parties enter into a contract. Hence,
those not allowed to be the object cannot also be a cause or consideration. If unlawful, contract
is void.

 Motive, even if unlawful, does not void a contract for it is any only intrinsic and not known to the
other party unlike cause or consideration.

Forms of Contracts

 General Rule:
Contracts shall be obligatory, in whatever form they may have been entered into, provided all
the essential requisites for their validity are present.
 Exceptions:

 When the law requires that a contract be in some form in order that it may be valid (Formal
or solemn contracts- example: wills)
 When the law requires that a contract be in some form in order that it may be enforceable
(Contracts covered by statute of fraud)

 Parties may compel each other to comply with the form required once the contract has been
perfected and partly executed.

Reformation of the Contract

 It is the remedy if there is a clear meeting of the minds between the parties but the written
contract or instrument failed to show their true intention due to fraud, accident, mistake or
inequitable conduct.

 Instances when there can be no reformation are:


 Simple unconditional donations inter vivos;
 Wills (after death)

1 - 8
REGULATORY FRAMEWORK FOR BUSINESS TRANSACTIONS R. LOPEZ

 When the agreement is void;

 If there is no meeting of minds between the parties, reformation cannot be availed of, the
remedy is annulment or an action of declaration of nullity of a void contract

Rules in the Interpretation of Contracts if there is a Doubt

 Words prevail over numbers


 Written words prevail over printed words
 Interpretation rules shall not favor the party who caused the ambiguity
 If still doubtful:
 In onerous contracts, interpretation rules should be for the greatest reciprocity of rights &
interest of both parties
 In gratuitous contracts, interpretation rules should be for the least transfer of right or
interest in favor of the transferor or donor.
 If doubt is in the principal object, the contract is void.

Defective Contracts

 Rescissible contract
 Voidable contracts also called annullable
 Unenforceable contracts also called validable
 Void contracts also called void ab ibnitio or null and void; inexistent

The following contracts are valid but may be rescinded (Rescissible contracts)
 Those entered into by guardians where the ward suffers lesion of more than ¼ of the value of
the things which are objects thereof;
 Those agreed upon in representation of absentees, if the latter suffer lesion by more than ¼ of
the value of the things which are subject thereof;
 Those undertaken in fraud of creditors when the latter cannot in any manner claim what are
due them;
 Those which refer to things under litigation if they have been entered into by the defendant
without the knowledge and approval of the litigations and the court;
 Payments made in a state of insolvency on account of obligations not yet enforceable; and
 All other contracts especially declared by law to be subject to rescission.

The following must be established in rescinding a contract (as a secondary remedy):


 The contract must be rescissible
 The party asking for rescission must have no other legal means to obtain reparation for the
damages suffered by him
 The person demanding rescission must be ready to return whatever he may be obliged to
restore if rescission is granted
 No innocent 3rd person is affected
 It must be brought within the 4-year prescriptive period.

The following contracts are valid but may be annulled or voided:


 Those where one of the parties is incapable of giving consent to a contract
 Those where the consent is vitiated by mistake, violence, intimidation, undue influence or fraud

 For the first contract aforementioned- annul within 4 years from the time incapacity ceases.
 For the second contract aforementioned- annul within 4 years from the discovery of fraud
or mistake or annulment within 4 years from the time the intimidation undue influence or
violence ceased
The following contracts are valid but cannot be enforced in the court if a party refuses to perform
(unenforceable contracts):
 Those where both parties are incapable of giving consent;
 Those entered into in the name of another by one without acting in excess of authority; and
 Those which do not comply with the Statute of Frauds.
The following agreements must be in writing otherwise they are not enforceable in courts (covered
by Statute of Fraud)

1 - 9
REGULATORY FRAMEWORK FOR BUSINESS TRANSACTIONS R. LOPEZ

 Agreements not to be performed within one year from the making thereof;
 Special promise to answer for the debt, default or miscarriage of another;
 Agreement in consideration of marriage other than mutual promise to marry;
 Agreement for the sale of goods, etc. at a price not less than P500,000;
 Contracts of lease for a period longer than one year;
 Agreements for the sale of real property or interest therein; and
 Representation as to the credit of a third person.
The following contracts are void from the very beginning, hence no right accrues.
 Those whose cause, object or purpose is contrary to law, morals good customs, public order or
public policy;
 Those which are absolutely simulated or fictitious;
 Those whose object is outside the commerce of men;
 Those whose cause or object did not exist at the time of the transaction
 Those which contemplate an impossible service;
 Those where the intention of the parties relative to the principal object of the contract cannot
be ascertained; and
 Those expressly prohibited or declared void by law.

Principle of Pari Delicto

 General Rule
When the defect of a void contract consists in the illegality of the cause or object of the
contract and both of the parties are at fault or in pari delicto, the law does not allow remedy
and leaves them where they are, under the maxim, “he who comes to court must come with
clean hands.”

 But the innocent party can recover what he has given in the 2nd to 7th contracts (preceding the
general rule) above and also in the 1st and 7th contract if not executed.

 Innocent third party can also ask for damages from the offenders if he was directly affected.

Other Concepts regarding Contracts

 An absolutely simulated contract is void since the parties have no intention at all to enter into a
contract.

 But a relatively simulated contract is valid (form is different from substance) since the parties
merely concealed their true agreement; They will be governed by their true agreement
provided it is lawful and not prejudicial to third persons as when they entered into a contract of
sale (in form) when their true intention is donation (substance).

“The most difficult thing in the world is to know how to do a thing and to watch someone else
do it wrong without comment.”

Theodore H. White

1 - 10
REGULATORY FRAMEWORK FOR BUSINESS TRANSACTIONS R. LOPEZ

OBLIGATIONS AND CONTRACTS QUIZZER

Instruction: Select the best answer to each of the following questions.


1
. The stipulation in a contract to the effect that the debtor should remain as a servant in the house
and in the service of her creditor so long as she had not paid her debt is void because it is:
a. Contrary to good customs.
b. Contrary to public policy.
c. Contrary to law and morality.
d. Contrary to obligations of contracts rule.
2
. Consent is manifested by the meeting of the offer and the acceptance upon the thing and the
cause which are to constitute a contract. Which of the following constitutes an offer?
a. An offer made through an agent.
b. Business advertisement of things for sale.
c. Advertisement for bidders.
d. Policitation.
3
. When one of the parties to a contract is compelled to give his consent by a reasonable and a
well-grounded fear of an imminent and grave evil upon his person or property, or upon the
person or property of his spouse, descendants or ascendants, there is:
a. Violence
b. Intimidation
c. Undue influence
d. Reverential fear
4
. Simulation of a contract may be absolute or relative. It is relative when:
a. The parties do not intend to be bound at all.
b. The contract is void.
c. The parties conceal their true agreement.
d. The intention of the parties is uncertain.
5
. The proper remedy is annulment of contract and not reformation when:
a. Mistake, fraud, inequitable conduct, or accident has prevented a meeting of the minds of the
parties.
b. A mutual mistake of the parties causes the failure of the instrument to disclose their real
agreement.
c. One party was mistaken and the other knew or believed that the instrument did not state their
real agreement, but concealed the fact from the owner.
d. The parties concealed their true agreement.
6
. The action to annul a voidable contract, such as a contract where one of the parties is incapable
of giving consent to the contract, is extinguished by:
a. Novation
b. Rescission
c. Ratification
d. Prescription
7
. When delivery takes place by the mere consent or arrangement of contracting parties as when
the vendor merely points to the thing sold which shall thereafter be at the control and disposal of
the vendee if the thing sold cannot be transferred to the possession of the vendee at the time of
the sale, delivery is effected
a. By traditio symbolica.
b. By traditio longa manu.
c. By traditio brevi manu.
d. By traditio constitution possessorium
8
. The obligation of the employer to pay death benefits and funeral expenses for his employee’s
death while in the course of employment as sanctioned by the Workmen’s Compensation Act is
one that arises from:
a. Law
b. Contracts

1 - 11
REGULATORY FRAMEWORK FOR BUSINESS TRANSACTIONS R. LOPEZ

c. Quasi-contracts
d. Quasi-delicts
9
.
Unless the law or stipulation of the parties requires another standard of care, the obligation to
give a thing carries with it the obligation to take care of it with:
a. Extra-ordinary diligence.
b. Ordinary diligence.
c. Diligence of a good father of a family.
d. Proper diligence.
10
. Mr. Montecarlo entered into a contract with Mr. Villacarlos by which Mr. Montecarlo promised to
deliver 1,000 cases of glasswares of the class and at the price stipulated in the contract. Such
delivery was to be made during the months of February and March 2006 to be used as gift items
in the reunions of their family and their classmates before the end of the said months
respectively. In this case no further demand or notice by Mr. Villacarlos on Mr. Montecarlo was
necessary because:
a. Time is of the essence of the contract
b. The obligation expressly so provides
c. The demand would be useless
d. Law so provides
11
. A borrower who uses the thing for a purpose different from that intended, delays its return,
receives the thing under appraisal, lend it to a third person, or saves his property instead of the
thing borrowed shall be liable even in case of fortuitous event, because:
a. The nature of the obligation requires the assumption of risk.
b. The parties have expressly stipulated such liabilities.
c. The law expressly so provides.
d. The party is guilty of delict or wrongful act.
12
. When the debtor binds himself to pay when his means permit him to do so, the obligation is
considered one:
a. With a condition dependent upon the debtor’s sole will.
b. With a period.
c. That is void.
d. With a resolutory condition.
13
. When two persons in their own right are reciprocally creditors and debtors of each other and
extinguishes both debts to the concurrent amount, what takes place is known as:
a. Compensation
b. Novation
c. Confusion or merger
d. Remission
14
. If the creditor to whom tender of payments has been made refuses without just cause to accept
it, the debtor shall be released from responsibility by:
a. Assignment of property.
b. Consignation of the thing or sum due.
c. Adjudication or dacion en pago.
d. Cession
15
. A executes a promissory notes in favor of B and the promissory note is negotiated by B and
subsequently is indorsed in favor of A. The obligation to pay the promissory note is thereby
extinguished because there is:
a. Confusion or merger
b. Novation
c. Remission
d. Condonation
16
. Compensation shall take place when:
a. One of the debts arises from the depositum or from the obligations of a depository or of a
bailee in commodatum.
b. One of the debts consists in civil liability arising from a penal offense.
c. Two persons, in their own right, are creditors and debtors of each other.

1 - 12
REGULATORY FRAMEWORK FOR BUSINESS TRANSACTIONS R. LOPEZ

d. Both parties are liable to each other involving two different things.
17
. The distinction between conventional subrogation and assignment is that in conventional
subrogation:
a. It is a mere cession of right.
b. An obligation is extinguished and another appears.
c. The same obligation, without being extinguished, is transferred to another.
d. It is also similar to expromision.
18
. The distinction between a chattel mortgage and a pledge is that in chattel mortgage:
a. The delivery of the personal property is necessary.
b. The registration of the property in the Registry of Property is not necessary.
c. The excess over the amount due after foreclosure goes to the debtor.
d. Property involved can be sold even without creditor’s consent.
19
. Mr. Renato Tal-od owns a mango tree bearing fruits, ready for harvest. He sells all the fruits of
that tree to Mr. Marcelo Abalos who pays Tala-od the sum of P5,000. Tala-od tells Abalos that he
can just harvest the fruits anytime he likes pointing at the particular tree. For legal purposes,
Tala-od has fulfilled his obligation to deliver mango fruits to Abalos by:
a. Traditio brevi manu
b. Traditio symbolica
c. Traditio longa manu
d. Traditio
20
. A promissory note signed by Martiniano and dated March 15, 2006 is worded as follows: “I
promise to pay Juanita the sum of Fifty Thousand Pesos (P50,000.), provided that if she should
fail in the October, 2006 CPA Examination, she shall return to me said amount”. The above note
gives rise to an obligation with:
a. Suspensive condition.
b. Casual condition.
c. Resolutory condition.
d. Conditions precedent.
21
. Bertulfo and Claudio promise to deliver a particular car valued at P100,000 to Manuela on
or before September 15, 2006. September 15, 2006 came and upon demand by Manuel for
delivery from Bertulfo and Claudio, Bertulfo was willing to deliver but Claudio refuse to deliver.
In the case at bar:
a. An action for a specific performance will lie against both Bertulfo and Claudio.
b. Both Bertulfo and Claudio shall be liable for P50,000.00 each with damages.
c. Bertulfo shall be liable for P50,000.00 without damages and Claudio shall be liable for
P50,000.00 and damages.
d. Ask a 3rd person to perform at the expense of both debtors plus damages.
22
. Carlito, Alfredo and Francis owed in solidum P50,000.00 to Ricardo as evidenced by a
promissory note due on September 30, 1995. The note prescribed on September 30, 2005. On
October 10, 2005, Carlito paid Ricardo. In this case, Carlito is:
a. entitled to collect P5,000.00 each from Alfredo and Francis.
b. not entitled to reimbursement from his co-debtor for the shares of the latter.
c. entitled to recover from Ricardo.
d. Entitled to recover the share of his co debtors plus interest.
23
. Cecilio had Eduardo kidnapped and tortured for refusing to sell his (Eduardo’s) land to Cecilio.
Eduardo who could no longer bear the physical pains inflicted upon him signed a document of
sale in favor of Cecilio. The sale is:
a. Void.
b. Voidable.
c. Valid.
d. Rescissible
24
. Carlo and Cleo are solidary debtors of Corrina, Luis, Edward, and Abelito joint creditors in the
amount of P20,000. 00. How much can Corrina collect from Carlo?
a. Corrina could collect P2,500.00 from Carlo.

1 - 13
REGULATORY FRAMEWORK FOR BUSINESS TRANSACTIONS R. LOPEZ

b. Corrina could collect P10,000.00 from Carlo. Corrina is then obliged to give P2,500.00
each to Luis, Edward and Abelito.
c. Corrina could collect P5,000.00 from Carlo.
d. Corrina could collect the whole P20,000.00 from Carlo but will in turn give P5,000.00 each
to Luis, Edward and Abelito.
25
. P.A.U. sold to R.D.E. the former’s car on April 1, 2006 for P300,000. P.A.U. promised to deliver
the car on April 15, 2006 but R.D.E. did not make any promise as to when to pay.
a. P.A.U. on April 15, 2006 can demand payment from R.D.E. because the obligation is not
subject to any condition.
b. R.D.E. will be obliged to pay only at or after delivery.

c. P.A.U. cannot demand payment until he delivers the car.


d. R.D.E can compel P.A.U to deliver the car before April 15, 2006 upon payment of the
selling price.
26
. Choose the contracts which are voidable:
a. Those undertaken in fraud of creditors when the latter cannot in any manner collect the
claims due them.
b. Those where the consent is vitiated by mistake, violence, intimidation, undue influence or
fraud.
c. Those whose object is outside the commerce of man.
d. Those where both parties are incapable of giving consent to a contract.
27
. A contract is in the stage of conception when:
a. There is meeting of the mind.
b. Negotiations are in progress.
c. The parties come to an agreement.
d. The contract is perfected.
28
. Mr. Debtor owes Mr. Creditor who has two (2) legitimate emancipated children, P50,000.00
payable on December 31, 2006. Which statement is correct?
a. If Mr. Debtor dies before December 31, 2006, Mr. Creditor cannot collect from the heirs of
Mr. Debtor.
b. If Mr. Debtor dies before December 31, 2006, Mr. Creditor can collect from the heirs of Mr.
Debtor.
c. If Mr. Creditor dies, his two (2) legitimate emancipated children cannot recover from Mr.
Debtor his obligation.
d. If both Mr. Debtor and Mr. Creditor die, the heirs of Mr. Creditor can collect from the heirs of
Mr. Debtor.
29
. In an obligation where only one 1) prestation has been agreed upon, but to extinguish the
obligation the debtor is allowed and does render another substitute, the obligation is:
a. Facultative Obligation.
b. Simple Obligation.
c. Alternative Obligation.
d. Conjoint Obligation.
30
. In a joint obligation, A, B, and C are debtors of joint creditors, D, E, and F in the amount of
P180,000.00. A’s obligation is:
a. Pay D P60,000.00.
b. Pay D, E, and F P180,000.00.
c. Pay D 120,000.00.
d. Pay D P20,000.00.
31
. On June 5, 2006, Jose Dizon of Angeles City is obliged to give to Ruben Samia, Jose Dizon’s
red crew cab. There was no delivery until June 15, 2006 when the garage of the red crew cab
collapsed due to heavy ash and sand spewed by Mt. Pinatubo and red crew cab was totally
destroyed. After the crew cab was destroyed and lost, is Jose Dizon still liable?
a. No. Even if Jose Dizon was already in default, he could plead impossibility of performance.
b. Yes. The obligation to deliver the crew cab is changed to pay the equivalent value because
Jose Dizon is in legal delay.

1 - 14
REGULATORY FRAMEWORK FOR BUSINESS TRANSACTIONS R. LOPEZ

c. No. Because there was no demand by Ruben Samia to deliver the crew cab and the
specific object was lost due to fortuitous event. The obligation is extinguished.
d. Yes. Because the contract is perfected.
32
. Mr. A Santos signed a letter addressed and delivered to Mr. S. Aguas. The terms of the letter
are:

1. An offer to sell a 500 sq. m. lot for P300,000.00.


2. An option time up to October 31, 2006 for Mr. S. Aguas to raise the P300,000.
3. Upon payment of the P300,000 Mr. Santos will execute and sign a
Deed of Sale.

On October 31, 2006, Mr. Santos sent a letter to Mr. S. Aguas asking for a new price of
P350,000.00 for the 500 sq. m. lot.

Can Mr. S. Aguas compel Mr. A. Santos to accept the P300,000.00 and make him sign and
execute a Deed of sale?
a. No. Mr. S. Aguas did not accept the offer of Mr. A. Santos.
b. Yes. Mr. A. Santos is already estopped by his signed letter and offer.
c. Yes. There was actual meeting of the minds.
d. Yes. The contract is perfected.
33
. Three (3) of the following contracts are void. Which is not?
a. Contract in writing contemplating and asking for a possible service.
b. Oral authority given to an agent in a sale of land.
c. Oral partnership agreement where immovable property is contributed.
d. Oral partnership agreement when capital is more than P3,000.00.
34
. A mortgaged his residential land to B as a guarantee for the payment of P400,000.00
obligation of A. They agreed that A should not sell the land while the obligation exists. Before
the maturity of the mortgage, C offered to buy the land from A.
a. A cannot sell the land to C because of the agreement not to sell.
b. A can sell the land to C only if B consents in writing.
c. A can sell the land to C despite the agreement not to sell.
d. A cannot sell the land to C unless A pays the obligation.
35
. If the obligor binds himself to perform his obligation as soon as “he shall have obtained a loan”
from a certain bank, this obligation is:
a. With a term
b. Conditional
c. Suspensive
d. Resolutory
36
. Mr. Esguerra, a former government employee, suffered from severe paranoia and was
confined in the mental hospital in 1998. After his release he was placed under the
guardianship of his wife to enable him to get his retirement pay. In 2005 he became a mining
prospector and sold some mining claims. In 2008 he sued to annul the sale claiming that he
was not mentally capacitated at the time of sale. The sale in question was:
a. Illegal.
b. Void.
c. Voidable.
d. Valid.
37
. Contracts entered into in a state of drunkenness or during a hypnotic spell are:
a. Void.
b. Valid.
c. Voidable.
d. Legal.
38
. The P1,000-bills issued by the Central Bank and in circulation are considered:
a. Checks.
b. Bill of exchange.

1 - 15
REGULATORY FRAMEWORK FOR BUSINESS TRANSACTIONS R. LOPEZ

c. Legal tender.
d. Promissory note.
39
. One of the following statements is not true:
a. A creditor is not bound to accept a check in satisfaction of his demands, because a check
even if good when offered, does not meet the requirements of legal tender.
b. The obligation of the debtor who had agree to pay in dollars in a foreign bill of exchange,
shall be discharged in Philippine currency measured at the prevailing rate of exchange at
the time the obligation was incurred.
c. The Philippine peso bills when attempted to be exported, as when carried in excess of that
allowed by the CB regulation, may be deemed to have been taken out of domestic
circulation as legal tender and thus, treated as commodity.
d. The purchasing power or value of money or currency depends upon, can come into being,
can be created or brought about by a law enacted by the legislative department of the
Government.
40
. It is one of the essential elements of an obligation:
a. A maker or drawer.
b. An object or prestation.
c. A drawee.
d. Money.
41
. Reluctantly and against her good sense and judgment, Rosemarie entered into a contract for
the delivery of 5 tables to Corazon for the price of P15, 000.00. Contract is:
a. Void.
b. Voidable.
c. Unenforceable.
d. Valid.
42
. It takes place when the parties do not intend to be bound at all by their agreement.
a. Apparent contract.
b. Absolutely simulated contract.
c. Relatively simulated contract.
d. Deed of assignment.
43
. Rescission of contract can take place in this case:
a. When the thing which are the object of the contract are legally in the possession of third
person who acted in bad faith.
b. When he who demands rescission can return whatever he may be obliged to restore.
c. When the party seeking resolution can perform only as to part and to part as to remainder.
d. When the seller cannot return the installments paid to him by the buyer.

44
. It refers to a joint obligation:
a. One in which debtor is liable for the entire obligation, and each creditor is entitled to
demand the whole obligation.
b. One in which either one of the parties is indispensable and the other is not necessary.
c. One in which the obligation of one is a resolutory condition of the obligation of the other, the
non-fulfillment of which entitles the other party to rescind the contract.
d. One in which each of the debtors is liable only for a proportionate part of the debt and each
creditor is entitled only for a proportionate part of the credit.
45
. It presupposes not only that the obligor is able, ready, and willing, but more so, in the act of
performing his obligation.
a. Promissory note.
b. Tender of payment.
c. Bill of exchange.
d. Obligation to sell.
46
. It is not a source of liability which will entitle the injured party to damages:
a. Culpa Acquilliana or negligence committed in the performance of a spontaneous act.
b. MORA or delay.

1 - 16
REGULATORY FRAMEWORK FOR BUSINESS TRANSACTIONS R. LOPEZ

c. Dishonesty, malice, or bad faith in the performance of an existing valid obligation.


d. Contravention of the tenor of obligation.
47
. A intimidated B to marry A’s daughter. After a year B would like to file action for annulment but
could not do so because A was around to intimidate him. The marriage contract is:
a. Rescissible.
b. Voidable.
c. Void.
d. Unenforceable.
48
. O gets a loan of P100,000.00 from P which becomes due on November 23, 2006 and
mortgaged his house as security for the debt. On Sept. 22, 2006 the mortgaged house was
completely destroyed by fire. A week after, P demanded payment from O on Sept. 29, 2006. Is
P’s demand valid?
a. No, because the obligation is one with a definite period and the demand would be
prejudicial to the rights of the debtor.
b. No, the obligation is extinguished because the object of the obligation is lost through a
fortuitous event.
c. Yes, the debts become demandable because the period established is for the benefit of the
creditor.
d. Yes, the debt become demandable because the collateral was lost through a fortuitous
event.
49
. The following, except one, are the characteristics of void or inexistent contract. Which is the
exception?
a. The defense of illegality of the contract is available to third person whose interest is not
directly affected.
b. They are not subject to ratification.
c. The right to raise defense of illegality cannot be waived.
d. The action or defense for declaration of their nullity or in existence of the contract does not
prescribe.
50
. Jesus owes Maria P1, 000 on June 30, 2006.
Maria owes Jesus P600 due June 20, 2006.
Maria owes Jesus P400 due on June 30, 2006.
Maria owes Jose P1,000 due on June 30, 2006.

On June 25, 2006 Maria cannot pay Jose P1, 000 so she assigns to Jose her credit of
P1,000 against Jesus without the knowledge of Jesus. On July 2, 2006 Jose tries to collect
from Jesus the P1,000. How much can Jose compel Jesus to pay?
a. Zero.
b. P1, 000
c. P600
d. P400
51
. It is a contract wherein a person binds himself to render special service or to do something in
behalf of another with consent of the latter.
a. Contract of piece of work.
b. Contract of service.
c. Contract of agency.
d. Contract of exchange.
52
. There is a contract of sale
a. when goods are delivered to a distributor on consignment.
b. when the manufacturer delivers the goods to an agent where the former retains the
ownership and dictates the term of the sale.
c. when goods are delivered to a person on charge account.
d. when goods are delivered to an agent to be sold by him and the agent is not available to
the manufacturer of the goods.

1 - 17
REGULATORY FRAMEWORK FOR BUSINESS TRANSACTIONS R. LOPEZ

53
. A stipulation whereby the pledgee or mortgagee automatically becomes the owner of the thing
pledged or mortgaged:
a. Pactum commisorium.
b. Consolidation of ownership.
c. Conventional redemption.
d. Consignation.
54
. Which of the following statements is true and correct?
a. Pledge and mortgage are accessory contracts because they cannot exist by themselves.
b. In both pledge and mortgage the creditor is entitled to deficiency judgment.
c. Where an obligation is secured by a pledge or mortgage and it is not paid when due the
pledgee or mortgagee may dispose of the thing pledged or mortgaged if there is an
agreement to that effect between the parties.
d. Unless otherwise agreed upon by the parties, the sale of the mortgaged property
extinguishes in full the obligation of the mortgagor to the mortgagee.
55
. A condition which if imposed on an obligation will be disregarded and will therefore make the
obligation immediately demandable.
a. If Juan commits suicide.
b. If Juan passes the bar examination.
c. If Juan kills Pedro.
d. If Juan will not rise from the dead.
56
. “A sells to B his lot and house in the city if A decides to transfer and live in the countryside” is
an example of
a. Mixed condition.
b. Potestative condition.
c. Casual condition.
d. Resolutory condition.
57
. It is a mode of extinguishing an obligation when two persons in their own right are creditors
and debtors of each other.
a. Confusion.
b. Reformation.
c. Compensation.
d. Novation.

58
. X, a minor sold to Y his parcel of land for P100,000. From the proceeds of the sale, X
deposited P30,000 in the bank, spent for food, clothing and other personal expenses P20,000,
joined a tour abroad costing P20,000, lost in gambling P20,000 and loaned P10,000 to Z who
became insolvent. Upon reaching the age of 21 years, X files an action for annulment which
the court granted. Y was required to return the land to X and X was made to return to Y the
amount of
a. P30, 000
b. P40, 000
c. P80, 000
d. P70, 000
59
. Statement I – A contract whose cause or object did not exist at the time of the transaction is a
defective contract which cannot be remedied by providing a cause or object as the case may
be in the contract.
Statement II – The nullity of the accessory obligation of the penal clause does not carry with it
the nullity of the principal obligation which remains in force and demandable. Determine
whether:
a. Statement I is true but Statement II is false.
b. Both statements are false.
c. Statement I is false Statement II is true.
d. Both statements are true.
60
. The period when the minds of the seller and the buyer have met on the subject matter and
cause of the interest is

1 - 18
REGULATORY FRAMEWORK FOR BUSINESS TRANSACTIONS R. LOPEZ

a. Negotiation of the sale.


b. Consensus of the sale.
c. Consumption of the sale.
d. Perfection of the sale.
61
. One of the following is not a characteristic of pledge and mortgage
a. Accessory.
b. Consensual.
c. Gratuitous.
d. Onerous.
62
. A obliged himself to give B a car if B places among the top ten in the CPA Board Exam.
Subsequently, they agreed that A would give B the car if B merely passes the CPA Board. This
is an example of:
a. Mixed novation.
b. Real novation.
c. Implied novation.
d. Personal novation.
63
. It is a mode of extinguishing an obligation when two persons in their own right are creditors of
each other:
a. Reformation.
b. Novation.
c. Compensation.
d. Confusion.
64
. In order that fraud may make a contract voidable:
a. It may be incidental but both parties should not be in pari delicto.
b. It may be serious and the parties must be in pari delicto.
c. It may be incidental but should have been employed by both parties.
d. It should be serious and should have not been employed by both contracting parties.
65
. One is not a requisite needed in order that the obligation shall be extinguished by loss or
destruction of the thing due:
a. When the thing is lost without the fault of the debtor.
b. When the thing lost is generic.
c. When the thing is lost before the debtor has incurred in delay.
d. When the thing lost is specific.
66
. X obligated himself to pay Y the amount of P30,000 30 days after May 31, 2007 plus a penalty
of P3,000 if he fails to pay the obligation on due date. After demand for payment by Y, X
offered to pay on July 30, 2007. Y can demand from X:
a. P30, 000 plus P3, 000 plus legal interest.
b. P30, 000 plus legal interest.
c. P30, 000 plus P3, 000.
d. P30, 000 plus P3, 000 plus legal interest plus damages.
67
. The following are requisites of pledge except:
a. The disposal of the property by the pledge.
b. Absolute ownership of the property pledged.
c. The thing pledged may be placed in the possession of a third person.
d. To bind a third person, it must be recorded in the office of the Registry of Deeds.
68
. The leviable properties of the debtor are transferred to the creditors to be sold and from the
proceeds, the creditors are paid:
a. Pledge
b. Cession
c. Dacion en pago
d. Chattel mortgage

1 - 19
REGULATORY FRAMEWORK FOR BUSINESS TRANSACTIONS R. LOPEZ

69
. A appoints B to sell his land.

Example I – If the authority of B is oral and B sells the land in writing. The sale is valid.
Example II – if the authority of B is in writing and B sells the land orally. The sale is valid.

Which of the following is correct?


a. Both examples are true.
b. Both examples are false.
c. First example is true but the second example is false.
d. First example is false but the second example is true.
70
. The obligation is demandable on the date of the obligation and shall continue to be in force up
to the arrival of the day certain:
a. Resolutory period.
b. Suspension period.
c. Indefinite period.
d. Legal period.
71
. A, B, and C secured a loan from D. The promissory note which evidence the obligation states:
“I promise to pay D or order P10,000 payable on demand” (Sgd.) A B C. The obligation is:
a. Solidary.
b. Divisible.
c. Indivisible.
d. Joint.
72
. Which one of the following is not an element of legal compensation?
a. Debts to be compensated are due and demandable.
b. There is controversy or adverse claim over any debts to be compensated.
c. There are two or more debts of the same kind.
d. There are two persons who are creditors or debtors of each other.
73
. X, Y, and Z joint debtors owe P18,000 to A, B, and C, solidary creditors. How much can B
collect from X?
a. P3, 000
b. P18, 000
c. P9, 000
d. P6, 000
74
. Which of the examples given below can legal compensation take place?
a. A owes B P1,000 due on June 30, 2007. B owes A P1,000 due on June 30, 2007 but C has
filed an adverse claim against A.
b. A owes B P1,000 payable on October 31, 2007. B owes A P1,000 due on October 31,
2007.
c. A owes B P1, 000 with C as his guarantor. B owes C P1,000.
d. A owes B P1,000 payable on May 31, 2007. B to deliver A 2 piculs of sugar worth P1,000
on May 31, 2007.
75
. X and Y are solidary debtors of A, B, and C, joint creditors to the amount of P30,000. How
much can B collect from X?
a. B can collect P30,000 from X. B in turn has to give A and C P10, 000 each.
b. B can collect P10, 000 from X.
c. B can collect P15,000 from X.
d. B can collect P30,000 from X. X in turn can recover from Y the amount of P15,000.
76
. Pledge and mortgage are accessory contracts. Which of the following statements is false?
a. They are meant to secure the fulfillment of a principal obligation.
b. They cannot exist if the principal obligation is void.
c. They can exist by themselves.
d. They can secure fulfillment of rescissible obligation.

1 - 20
REGULATORY FRAMEWORK FOR BUSINESS TRANSACTIONS R. LOPEZ

77
. Case 1 – A hired B for P10,000 to kidnap C, and he paid B in advance. Before B could kidnap
C, A relented and stopped B from performing the contract. The court may not allow A to recover
from B the P5,000 he paid in advance.

Case 2 – A at the point of a gun, compels B to marry him. Since the contract of marriage is
voidable either A or B has the right to file the action for annulment. Determine whether:
a. Both cases are false.
b. Case 1 is true but case 2 is false.
c. Case 1 is false but case 2 is true.
d. Both cases are true.
78
. ABC are solidary creditors of X for P10,000. A makes a will giving the P10,000 debt to X as
legacy. The obligation is extinguished by:
a. Confusion of debt.
b. Novation of debt.
c. Remission of debt.
d. Payment of debt.
79
. Which of the following is not a special form of payment?
a. Cession.
b. Application of payment.
c. Consignation.
d. Dacion in payment.
80
. Mr. Gamboa owes Mr. Evangelista P100,000. Mr. Gamboa knows that on maturity date, he will
not be able to pay Mr. Evangelista, and in order to prevent attachment of his property by Mr.
Evangelista, Mr. Gamboa, before maturity of his debt, executes a contract pretending to sell to
Mr. Sandiego his property. Which of the following statement is not correct?
a. The contract is not valid for lack of consideration.
b. The contract is binding between Mr. Gamboa and Mr. Sandiego.
c. The contract being simulated and executed to defraud Mr. Evangelista is void.
d. Mr. Evangelista can seek rescission of the fictitious contract.
81
. A owes solidary creditors XYZ P10, 000. There is remission of the debts when:
a. X borrows P10,000 from A.
b. X waives partially the obligation of P10,000 to A.
c. Z makes a will giving the P10,000 debt to A as legacy.
d. Y tells that instead of paying P10,000, A shall just deliver a ring to Y.
82
. Mr. Ayco offered to sell his land to Mr. Cruz for P30,000. Mr. Cruz accepted the offer and paid
Mr. Ayco the purchase price. Mr. Ayco delivered the owner’s duplicate of the Transfer
Certificate of title of the land. Mr. Cruz wants to register the land to his name but the Register
of Deeds asks Mr. Cruz for the Deed of Sale. What can Mr. Cruz do?
a. He may occupy and use the land as a buyer in good faith.
b. He cannot compel Mr. Ayco to return the payment because the contract is unenforceable.
c. He may compel Mr. Ayco to execute the Deed of Sale because the contract is valid.
d. He may sue Mr. Ayco to return the purchase price under the legal principle that no one may
enrich himself at the expense of another.
83
. Three of the following are essential requisites of contract of mortgage. Which one is not?
a. The person instituting the mortgage has the free disposal of his property.
b. The contract must be in writing.
c. The mortgagor is the absolute owner of the thing mortgaged.
d. The mortgage is constituted to secure the fulfillment of a principal obligation.
84
. In real mortgage, the following rules are valid, except one:
a. A stipulation in the mortgage contract prohibiting the owner from alienating the immovable
mortgage is valid.
b. The mortgagee may alienate the mortgage credit or assign to a third person in whole or in
part.
c. Any stipulation allowing the mortgage creditor to appropriate the property mortgaged is null
and void.
d. If alienation of mortgage credit is not registered, it is still valid between the parties.

1 - 21
REGULATORY FRAMEWORK FOR BUSINESS TRANSACTIONS R. LOPEZ

85
. When the characters of the creditor and the debtor are merged in one and the same person,
there is extinguishment of the obligation by:
a. Compensation.
b. Merger of rights.
c. Novation.
d. Remission.
86
. The process of intentionally deceiving others by producing the appearance of a contract which
is different from the true agreement is:
a. Relative simulation of a contract.
b. Absolute simulation of a contract.
c. Annulment of a contract.
d. Misrepresentation.
87
. There shall be no reformation of contract in the following cases, except:
a. In testamentary wills.
b. Simple donation intervivos with no condition being imposed.
c. Contract of mortgage executed as a contract of sale.
d. A contract of sale fraudulently made as a contract of lease.
88
. X and Y and Z solidarily owe A, B, C, and D, joint creditors P30, 000. How much can collect
from X?
a. P7, 500 only.
b. P20, 000, A in turn has to give P5, 000 to B, C, and D.
c. P10, 000, A in turn has to give P20, 000 each to B, C, and D.
d. P5, 000 only.
89
. A, B, and C solidarily bounds themselves to deliver to X a Honda Motorcycle. The obligation
was not fulfilled through the fault of A. thereupon, X filed an action in court against C and the
court awarded P36,000 to X. which of the following situation is valid?
a. If C pays X the P36,000, C can collect from A P24,000 and B P12,000. Later B can ask for
reimbursement from A P12,000.
b. X has to collect P12,000 each from A, B, and C to satisfy the court’s award of P36,000.
c. If C pays X the P36,000, C can collect from B P10,000 and from A P16,000.
d. X cannot collect the whole amount of P36,000 from C.
90
. Which of the following statements regarding contract of pledges is not correct?
a. A pledge cannot be constituted unless the thing pledged be placed in the possession of the
creditor
b. Pledge is a real contract because it is perfected by delivery of the thing pledged.
c. Pledge is an accessory contract because it needs a principal obligation to exist.
d. A pledge cannot be constituted to secure the performance of a voidable or unenforceable
or even a natural obligation.
91
. When the mortgage is due and remains unpaid, can the Mortgage appropriate the mortgaged
property?
1st Answer: No, the only right of the mortgagee-creditor is foreclosing the mortgage.
2nd Answer: Yes, if there is stipulation in the mortgage contract allowing the mortgage-creditor
to appropriate the property mortgaged.
a. 1st answer correct; 2nd answer wrong.
b. 1st answer wrong; 2nd answer correct.
c. Both answers are wrong.
d. Both answers are correct.
92
. Diaz is under obligation to give Cruz, at Diaz option, a car, a pick-up, or a van.
1st Statement: if the car and the pick-up were destroyed by Diaz own fault and later Diaz
communicated to Cruz to deliver the van, but got lost by a fortuitous event, Diaz should not be
held liable.
2nd Statement: if the car and the pick-up were destroyed by a fortuitous event and later the
van was lost by Diaz’ fault, Diaz should be liable.
a. 1st statement correct, 2nd statement wrong.
b. 1st statement wrong, 2nd statement correct.

1 - 22
REGULATORY FRAMEWORK FOR BUSINESS TRANSACTIONS R. LOPEZ

c. Both statements are correct.


d. Both statements are wrong.
93
. Through insidious words or machinations, A was able to induce B to enter into a contract which
without them B would not have agreed to it. There is:
a. Undue influence.
b. Fraud.
c. Mistake.
d. Misrepresentation.
94
. X owes Y P40,000. With the consent of both parties, Z pays Y P20,000 which makes Y and Z
creditors of X to the amount of P20,000 each. X becomes bankrupt and has P20,000 assets
left.
a. Z should get the P20,000.
b. Y and Z should divide the P20,000 equally.
c. X may choose whom to pay.
d. Y should get the P20,000.
95
. On July 30, 2007, Arrieta wrote Bascon a letter offering a contract of sale. On August 26,
Bascon wrote a letter of complete acceptance of the contract which was received by Arrieta on
August 31, 2007. But on August 30, 2007, Arrieta had already written Bascon a letter
withdrawing the offer which was received by Bascon on September 1, 2007. Is there a
perfected contract between Arrieta and Bascon?
1st Answer: there is a perfected contract between Arrieta and Bascon because the acceptance
was made before the withdrawal.
2nd Answer: there is a perfected contract because Arrieta came to know of the acceptance
before Bascon came to know of the withdrawal.
a. Both answers are correct.
b. 1st answer correct, 2nd answer wrong.
c. Both answers are wrong.
d. 1st answer wrong, 2nd answer correct.
96
. If the obligation of the debtor is “I will pay you my debt after I have arrived from abroad”, this is
a. Unenforceable.
b. With a period.
c. Void.
d. Conditional.
97
. Which of the following contracts is not unenforceable?
a. That were both parties are incapable of giving consent.
b. That were one party is incapable of giving consent.
c. That which is entered into the name of another person by one who has given no authority
or legal representation.
d. That which does not comply with the statute of frauds.
98
. X delivers 10 sets of television to A for the latter to sell them for P10,000 each. A 10%
commission is agreed upon the parties. One day A sells one TV set to a customer on credit but
for a price of P12,000. Suppose that X does not consent to the sale on credit?
a. X may demand P10,800 from A.
b. X may demand P10,000 from A.
c. X may consider the contract as unenforceable.
d. X may demand P9,000 from A.
99
. Cuenca mortgaged his lot and house to Bascon as collateral for the payment of his loan
obligation. The mortgage contract stipulates that Cuenca cannot sell the property while the
obligation exists. Before the maturity of the mortgage, Diaz offered to buy the property from
Cuenca.
a. Cuenca cannot sell the property to Diaz unless he pays the loan obligation.
b. Cuenca can sell the property only if Bascon consent to it.
c. Cuenca cannot sell the property to Diaz because of the agreement not to sell while the
mortgage exists.
d. Cuenca can sell the property to Diaz despite the stipulation in the mortgage contract not to
sell.

1 - 23
REGULATORY FRAMEWORK FOR BUSINESS TRANSACTIONS R. LOPEZ

100
. Santos sold his Toyota pick-up to Ramon for P250,000. No date was fixed for the performance
of the obligation by the seller and the buyer. The obligation of the seller is:
a. To rescind the sale because there is no time fixed for the performance of the seller and the
buyer.
b. To deliver the pick-up only after the buyer writes the seller demanding the delivery of the
pick-up.
c. To deliver the pick-up after the buyer performs the obligation.
d. To deliver the pick-up immediately because the sale is already perfected contract.
101
. Araneta wrote a letter to Bascon wherein he offered to sell a piece of land. In Araneta’s letter
he gives Bascon a period of two months within which to pay the price of P500,000. After 50
days Araneta told Bascon that he is increasing the price of the land to P700,000. Can Bascon
compel Araneta to accept the P500,000 first offered by Araneta and execute the deed of the
sale?
a. No, for Bascon did not signify his acceptance of the offer.
b. Yes, because the period of 2 months has not yet expired.
c. Yes, because Araneta is already estopped by his signed letter.
d. Yes, because there was actual meeting of minds of the parties.
102
. Arco mortgaged his land to Bascon as collateral for the payment of a loan obligation. The
mortgage contract provides that Arco cannot sell the land while the obligation exists. Before
maturity of the mortgage, Cruz offered to buy the land from Arco.
a. Arco cannot sell the land because of the stipulation in the contract.
b. Arco can sell the land despite the agreement not to sell.
c. Arco can sell the land only if Bascon consents in writing.
d. Arco cannot sell the land unless he pays the obligation.
103
. Chattel Mortgage as distinguished from pledge is
a. The delivery of the personal property is necessary.
b. The registration of the property in the Registry of Property is not necessary.
c. The excess over the amount due after foreclosure goes to the debtor.
d. The sale in auction of the object extinguishes the obligation.
104
. D pledged his watch to C for P20, 000. D failed to pay his obligation on time.
Case 1 – If C sold it at public auction for P18, 000, C cannot recover the deficiency even if
there is stipulation.
Case 2 – If C sold it at public auction for P22, 000, D can recover the excess if stipulated.
a. 1st wrong, 2nd correct.
b. 1st correct, 2nd wrong.
c. Both cases are correct.
d. Both cases are wrong.
105
. Which of the following is common requisite for Pledge, Chattel Mortgage and antichrises?
a. The amount of principal and interest must be in writing.
b. The property must be in possession of the creditor.
c. To bind third person they must be duly recorded in the office of Register of Deeds.
d. It is constituted to secure the fulfillment of a principal obligation.
106
. Which of the following statements is true and correct?
a. Unless otherwise agreed upon by the parties, the sale of the mortgaged property
extinguishes in full the obligation of the mortgagor to the mortgagee.
b. Pledge and mortgage are considered principal contract.
c. When the obligation is secured by a pledge or mortgage and it is not paid when due, the
pledgee or mortgagee may dispose the collateral even if there is no agreement to that
effect between the parties.
d. In both pledge and mortgage the creditor is entitled to deficiency judgment.
107
. Which of the following contracts is not void ab initio?
a. That whose object is outside the commerce of man.
b. That whose object did not exist at the time of transaction.
c. That which contemplates an impossible service.
d. That which is undertaken in fraud of creditors.

1 - 24
REGULATORY FRAMEWORK FOR BUSINESS TRANSACTIONS R. LOPEZ

108
. Contracts which cannot be sued upon unless ratified, thus it is if they have no effect yet are
a. Voidable.
b. Rescissible.
c. Void.
d. Unenforceable.
109
. Which of the following contracts is valid?
a. Oral contract of agency giving authority to an agent to sell the land belonging to the
principal.
b. Oral, partnership agreement where immovable property is contributed.
c. Oral contract of sale of an immovable property entered into by an agent who was given
authority orally by the principal.
d. Oral agreement to answer all expenses for the wedding reception if A marries B.
110
. A, B, and C solidarily owe X and Y P30, 000. X remitted the entire obligation in favor of A. The
effect is
a. The obligation is not extinguished until A collects from B and C.
b. The obligation is not yet extinguished until Y is paid by X of Y’s share of the credit.
c. A cannot recover from B and C because remission in his favor extend to the benefit of B
and C.
d. A can recover from B and C their respective share of the debt.
111
. When the debtor binds himself to pay his means permit him to do so, the obligation is
a. Conditional.
b. Pure.
c. Simple.
d. With a period.
112
. When A voluntarily takes charge of the neglected business of B without the latter’s authority
where reimbursement must be made for necessary and useful expenses, there is a
a. Quasi delict.
b. Quasi contract.
c. Negotiorum gestio.
d. Solutio indebiti.
113
. Which of the following is not a rule in the interpretation of contracts?
a. If some stipulation of any contract should admit of several meanings, it shall be
understood as bearing that import which is most adequate to render the contract
effectual.
b. Words which may have different significations shall be understood in that which is most in
keeping with the nature and object of the contract.
c. In order to judge the intention of the contracting parties, their contemporaneous and
subsequent acts shall be principally considered.
d. Although the cause is not stated in the contract, it is presumed that it exists and is lawful,
unless the debtor proves the contract.
114
. Contracts are effective and binding only between the parties, their assigns and their heirs.
Three of the following enumerations are exceptions as provided by law. Which does not
belong to the exceptions?
a. Where there is a stipulation in favor of a third party
b. Where one of the parties to the contract dies and thereafter a suit is filed on the basis of
the contract
c. Where the obligations arising from contract are not transmissible by their nature
d. Where the obligations arising from the contract are not transmissible by stipulation or by
provision of law.
115
. A solidary obligation is one in which each of the debtors is liable for entire obligation or debt,
and each of the creditors is entitled to the entire credit. Obligations shall also be considered
solidary under the three following exceptions. Which does not belong to the exception?
a. When solidarity is expressly stipulated in the obligation
b. When the prestation is indivisible and there are two or more debtors and creditors
c. When the law expressly provides solidarity

1 - 25
REGULATORY FRAMEWORK FOR BUSINESS TRANSACTIONS R. LOPEZ

d. When solidarity is required from the nature of the obligation


116
. In order that fraud may make a contract voidable
a. It may be incidental but should have been employed by both parties
b. It should be serious and the parties must be in pari delicto
c. It should be serious and should not have been employed by both contracting parties
d. It may be incidental but both parties should not be in pari delicto
117
. Payment of the obligation by a solidary debtor shall not entitle him to reimbursement from his
co-debtors.
a. If such payment was made before the obligation is due
b. If such payment was made after the obligation has prescribed or become illegal
c. If such payment was made in compliance with the demand made to him by all the
creditors
d. If such payment was made after the obligation has become due and demandable and
notice of payment was made only to him.
118
. A and B who are both unemancipated minors entered into a contract. The contract entered
into by and between them is
a. Rescissible
b. Unenforceable
c. Voidable
d. Void
119
. A, B and C secured a loan from X. The promissory note which evidences the obligation
states: “I promise to pay” and signed by A, B and C. The obligation is:
a. Joint
b. Solidary
c. Divisible
d. Indivisible
120
. Which of the following contracts cannot be ratified?
a. Those whose cause or object did not exist at the time of the transaction
b. Unauthorized contracts
c. Those where both parties are incapable of giving consent
d. Those that fail to comply with the Statute of Frauds
121
. Consignation is a mode of payment which extinguishes an obligation. Which of the following
is not a requisite for consignation?
a. Actual consignation with proper judicial authorities
b. Prior tender has not been made
c. Existence of a valid debt
d. There must be prior notice of consignation to persons interested in the fulfillment of the
obligation.
122
. The creditor shall have the right to indemnity for damages when, through the fault of the
debtor, all the things which are alternatively the object of the obligation have been lost or
compliance of the obligation has become impossible. The indemnity shall be fixed taking as a
basis
a. The value of the least expensive thing
b. The value of the most expensive thing
c. The value of the last thing which disappeared
d. The value of the first thing which disappeared.
123
. A, B and C are solidary debtors of X in the amount of P12,000. X then made a demand from
A but was able to collect P8,000 only because X was remitting A’s share of P20,000. How
much can A collect from B and C?
a. P12,000
b. P8,000 each
c. P6,000 each
d. P4,000 each

1 - 26
REGULATORY FRAMEWORK FOR BUSINESS TRANSACTIONS R. LOPEZ

124
. A is indebted to B for P20,000. X is the guarantor of A. B is also indebted to A for P8,000.
How much will X be liable as guarantor if B sues A, and A cannot pay?
a. P12,000
b. P20,000
c. P8,000
d. X has no liability.
125
. On October 4, 1987, A is indebted to B for P50,000 for a 20-day period. A proposed to B that
X will pay A’s debt, and that A will be free from all liabilities. B and X agree to the proposal.
On October 25, 1987, when B tries to collect from X, he finds that X is insolvent. At the time
of delegation, X was already insolvent but this was not known to A. The insolvency is not of
public knowledge. So B sues A on the ground that it was A who made the proposal and that A
really guaranteed X’s solvency. Decide.
a. A is liable because he is presumed to have guaranteed X’s solvency.
b. A is not liable, because A does not know the insolvency of X at the time of delegation and
neither was the solvency of public knowledge.
c. A is liable because he did not exercise due diligence in determining the insolvency of X.
d. A is liable because X agree to the proposal to make himself solidarily liable for the
obligation.
126
. A and B are creditors mancomunados of X and Y, joint debtors, for a total sum of P9,000. A
owns 1/3 of the credit; B owns 2/3 of it. But X owes 2/5 of the debts; Y owes 3/5 of the debt.
In this case,
a. B can only collect from X, P5,400 and from Y P3,600.
b. A can only collect from X, P4,500 and from Y, P4,500
c. A can only collect from X, P3,600 and from Y, P5,400
d. B can only collect from X, P9,000 and from Y, P-0-
127
. A guardian of B sold B’s house and lot worth P480,000 for P240,000.
a. The contract can be rescinded because of inadequacy of price.
b. The contract cannot be rescinded because there is no fraud, mistake or undue influence.
c. The contract cannot be rescinded because all the elements of contract are present
d. The contract cannot be rescinded because it is expressly provided by law as one of the
contracts that cannot be rescinded.
128
. X entered into a contract with Y, whereby X sold his land orally to Y. The land has been
delivered and the money has been paid. Is the oral sale of land valid?
a. The contract is not valid because it is not in writing as required by the Statute of Frauds.
b. The contract is not valid because the contract is not made in public instrument
c. The contract is unenforceable
d. The contract is valid because the contract is already perfected and executed.
129
. To defraud his creditors, A contracted B by selling a land to B. B now seeks to register the
land with the Register of Deeds. X a creditor of A seeks to prevent the registration on the
ground that the contract is rescindable. Despite X’s objection, may the land be registered
based on the contract in B’s name?
a. The land cannot be registered based on the contract because the contract is rescindable.
b. The land cannot be registered because the contract is in fraud of creditor.
c. The land can be registered based on the contract because the contract is not yet
rescinded.
d. The land can be registered because the contract is valid and can be attacked collaterally
in a land registration proceeding.
130
. On July 15, 1986, X entered into a contract with Y. On February 10, 1987, X discovered that
fraud was committed at the time he entered into the contract, a fraud that vitiated his
consent. The action for annulment shall be brought,
a. Within three years from the time of the fraud
b. Within four years from February 10, 1987
c. Within four years from the time A entered into the contract
d. On February 10, 1987

1 - 27
REGULATORY FRAMEWORK FOR BUSINESS TRANSACTIONS R. LOPEZ

131
. On September 1, 1987, A entered into a contract with B, whereby A sells to B 5,000 sacks of
sugar to be delivered on the 15th and to be paid in full on the 30th. There was no agreement
for rescission based on non-payment. A did not deliver on the 15th, but on the 30th he was
willing and offering to deliver. But B did not make payment on said date, and so A did not like
and refused to make the delivery. Decide.
a. A cannot rescind the contract for non-payment of the price.
b. A cannot refuse to deliver the goods.
c. B is not entitled to recover damages.
d. A can rescind the contract for non-payment of the price.
132
. X alleged that Y promised to give X one hectare of land. This is in consideration of X’s
meritorious service to Y. Y pleads in defense that since the promise was not in writing, it is
unenforceable under the Statute of Frauds. Decide.
a. The promise is unenforceable because it is not in writing.
b. The Statute of Frauds is applied because A has rendered service already.
c. The Statute of Frauds is inapplicable here, because the promise to give the land is not a
sale of real property.
d. The Statute of Frauds can apply to partially executed contract.
133
. A has a daughter B; X has a son Y. A, B, X and Y agree together that Y will marry B. The
agreement is oral. If B later on refuses to marry Y who has spent for the necessary wedding
preparations, and X and Y bring an action against A and B, will the action prosper? Decide.
a. Between Y and B, the action will not prosper because the agreement is made orally.
b. In case of A and X, the action will prosper because the agreement which was oral is
enforceable as it is based on the consideration of marriage.
c. As to A and X, the action will not prosper because the agreement is not enforceable as it
was not they who mutually promised to marry each other.
d. The action of X and Y against A and B will prosper because the agreement is based on
the consideration of marriage, other than a mutual promise to marry.
134
. X and Y are solidary debtors of A, B, C, and D, joint creditors to the amount of P8,000.00.
How much can A collect from X?
a. A could recover P4,000 from X. A, in turn has to give to B, C, and D P1,000 each.
b. A could recover P2,000 only from X
c. A could recover P1,000 only from X.
d. A could recover P8,000 from X. A, in turn has to give to B, C, and D P2,000 each.
135
. Who is liable for the loss of the subject matter by fortuitous event?
a. Creditor
b. Debtor
c. Both creditor and debtor
d. None of them
136
. Mr. ABC is obliged to give Mr. XYZ his only car on July 15, 1987. Mr. ABC did not deliver the
car on July 15, 1987. On July 20, 1987 an earthquake destroyed the building where the car
was in garage and the car was destroyed. Is Mr. ABC still liable?
a. No. Considering that no demand to deliver was made by Mr. XYZ and the specific thing
was lost due to fortuitous event, the obligation is extinguished.
b. No. The obligation is extinguished, even if the debtor is already in default, because the
debtor can plead impossibility of performance.
c. Yes. Mr. ABC is already in legal delay, thus the obligation to deliver the lost specific thing
is converted into monetary claim for damages.
d. Yes. The creditor can instead demand for a substitute of equivalent value from the debtor.
137
. A and B are solidary debtors of X, Y, and Z , joint creditors to the amount of P15,000. How
much can Z collect from B?
a. Z could recover P7,500 from B.
b. Z could recover P5,000 from B.
c. Z could recover P15,000 from B. Z, in turn has to give X and Y P5,000 each.
d. Z could recover P15,000 from B. B in turn can collect from A the amount of P7,500.
138
. These persons are bound by contract:

1 - 28
REGULATORY FRAMEWORK FOR BUSINESS TRANSACTIONS R. LOPEZ

a. Contracting parties.
b. Heirs
c. Assigns or assign
d. All of them
139
. When two (2) persons are reciprocally debtors and creditors, there is:
a. Confusion
b. Compensation
c. Consignation
d. Novation
140
. A, B, and C borrowed P24,000 from Y and Z and signed a promissory note dated January
15, 1987 and due within six (6) months. How much can Y collect from A?
a. P12,000
b. P 4,000
c. P 8,000
d. P24,000
141
. A, B and C borrowed P36,000 from X and Y. The three (3) debtors signed a promissory note
on January 10, 1988 promising to pay the creditors on or before July 10, 1988. How much
can X collect from C?
a. P18,000
b. P6,000
c. P12,000
d. P36,000
142
. When the subject matter of a contract is lost through a fortuitous event, who is liable?
a. The debtor.
b. The creditor.
c. None of them.
d. Both creditor and debtor.
143
. OE is obliged to give OR his 1982 Mitsubishi, 4-door sedan Lancer car with plate number
NBA 123 on September 30, 1989. On October 10, 1989, OE did not yet deliver the car which
was totally destroyed by an earthquake on such date. Is OE still liable?
a. No. The obligation is extinguished. The specific thing was lost due to fortuitous event and
no demand to deliver was made by OR.
b. Yes, OE is in legal delay. OR can claim damages.
c. No. Even OE is already in default and can plead impossibility of performance.
d. Yes. OR can instead demand for another car or equivalent value from OE.
144
. Contracts entered into during lucid-interval are:
a. Unenforceable
b. Voidable
c. Void
d. None of the three (3).
145
. In three (3) of the following cases advanced payment by the debtor is not recoverable. Which
is the exception?
a. The obligation was not yet due and demandable but the debtor believed it was already
due and demandable.
b. The payment is only for interest and credited to the proper period.
c. The advanced payments were made by both parties reciprocally.
d. The debtor was not aware of the period.
146
. Three (3) of the following maybe valid objects of a contract, except:
a. All that are within the commerce of man.
b. All services which are not contrary to law.
c. Impossible things or services.
d. All rights that are transmissible.

1 - 29
REGULATORY FRAMEWORK FOR BUSINESS TRANSACTIONS R. LOPEZ

147
. A contract executed by two (2) parties and one (1) of the parties is not capable of giving
consent, the contract is:
a. Voidable
b. Rescissible
c. Void
d. Unenforceable
148
. Three (3) of the following contracts are void. Which one is not?
a. Oral contract of Partnership of three (3) partners and capital contribution is more than
P3,000 in cash.
b. Written contract contemplating impossible services.
c. Oral contract of Partnership where real estate is contributed as capital.
d. Agent’s authority to sell real property given orally.
149
. C is the creditor of D in the amount of P50,000. G is the guarantor of D. D paid C partially
with P20,000. A, not knowing the partial payment of D and against the will of D, paid C the
amount of P50,000. What is the effect of this payment in the obligation?
a. The obligation is extinguished. A cannot recover any amount from D, but A can demand
reimbursement from G in the amount of P50,000.
b. The obligation is extinguished. A can demand P30,000 from D because this amount
benefited D, or , A, having been subrogated into the rights of C, can proceed against
guarantor G.
c. The obligation is not extinguished. A’s payment being against the will of D does not
extinguish the obligation.
d. The obligation is extinguished. A can demand P30,000 from D, but if D cannot pay, A
cannot ordinarily proceed against guarantor G because A is not entitled to subrogation.
150
. Mr. Ong leased to MR. Santos a 5 KVA generator for two (2) years at a lease rental fee of
P2,000 per month and signed an option in favor of Mr. Santos to buy the generator at the end
of the term of the lease of P60,000. All rental fees paid are to be considered as partial
payment of the sale. After 12 months Mr. Santos was able to pay the rental fees for 9 months
and was in arrear for the 3 months rental fees. MR. Ong terminated the lease contract and
repossessed the generator. The consequence of the transaction is:
a. Mr. Ong can collect the rental fees for three (3) months which are in arrears.
b. Mr, Ong can collect the rental fees for the unexpired 12 months of the lease contract.
c. When Mr. Ong took possession of the generator, he has no further action against Mr.
Santos.
d. Mr. Ong, in terminating the lease and repossesing the generator is obliged to refund the 9
months rental fees paid by Mr. Santos.
151
. D pledged his Singer Sewing Machine to C for P8,000. D was unable to pay the obligation 60
days after due date. C sold the machine at public auction for P6,000.
a. C cannot recover the deficiency of P2,000 even if there is stipulation that he can.
b. C can recover the deficiency of P2,000 even without stipulation.
c. C cannot recover the deficiency of P2,000.
d. C can recover the deficiency of P2,000.
152
. It is one of the essential elements of an obligation
a. A maker or drawer
b. An object or prestation
c. A drawee
d. Money
153
. Three of the following are requisites of an obligation. Which is the exception?
a. Prestation
b. Efficient cause
c. Delivery
d. Passive subject
154
. The obligation of the employer to pay death benefits and funeral expenses for his employee’s
death while in the course of employment as sanctioned by the Workmen’s Compensation Act
is one that arises from

1 - 30
REGULATORY FRAMEWORK FOR BUSINESS TRANSACTIONS R. LOPEZ

a. Law
b. Contracts
c. Quasi-contracts
d. Answer not given
155
. In the execution of obligations, liability for malice or bad faith
a. Extends not only to results intended but also to their foreseen consequences
b. Is demandable in all obligations but may be renounced in advance.
c. Extends only to results intended but excluding exemplary damage.
d. Extends to natural consequences even if they exceed the debtor’s expectations.
156
. Thru some mistake on the part of bank personnel, A was given P1,000,000 instead of only
P1,000. This situation will be governed by the provision of the law on
a. Contracts
b. Obligations
c. Quasi-delicts
d. Qasi-contracts
157
. When A voluntarily takes charge of the neglected business of B without the latter’s authority
where reimbursements must be made for necessary and useful expenses, there is a
a. Quasi-delict
b. Quasi-contract
c. Negotiorum gestio
d. Solutio indebiti
158
. All are requisites of negotiorum gestio except:
a. A property or business is neglected
b. A third person voluntarily manages it
c. Owner is benefited
d. Implied authority is given to the officious manager.
159
. Which is not a requisite of solutio indebiti?
a. There is a valid obligation
b. It is due and demandable
c. Debtor pays by mistake
d. Creditor accepts the payment aware that it is not yet due
160
. All are sources of obligations except:
a. Du ut des agreement
b. Tax Code
c. A debtor voluntarily paying his obligations that has prescribed
d. A driver’s negligence resulting to physical injury to his passengers.
161
. All of the following are obligations of the debtor if the object is a determinate thing.
a. To deliver the specific thing promised
b. To deliver the accession or its accessories even though not agreed upon
c. To deliver the fruits from the perfection of contract
d. To take care of the thing promised pending delivery with diligence required by law
162
. Which is not a source of liability to pay damages?
a. Culpa contractual
b. Fraud in inducement
c. Failure to deliver on the date agreed by the parties
d. Obligor violating the tenor of their agreement with the obligee.
163
. All are remedies of the obligee in case of breach of obligation to deliver a generic thing,
except:
a. Right to compel specific performance
b. Right to ask a 3rd person to do it at debtor’s expense
c. Right to convert the obligation to its monetary value
d. Right to ask for other damages

1 - 31
REGULATORY FRAMEWORK FOR BUSINESS TRANSACTIONS R. LOPEZ

164
. A sold to B 100 baskets of lanzones at P50 a basket. Thereafter, A can deliver only 75
baskets and offered this number to B and no more, but at P55 each.
a. B can refuse to accept delivery of the 75 baskets without liability.
b. B must pay for 100 baskets but at P50 each.
c. B can accept 75 baskets but pay P55 for each
d. A can require B to accept 100 baskets at P55 each.
165
. Mr. Montecillo entered into a contract with Mr. Villarojo by which Mr. Montecillo promised to
deliver without need of any reminder from Mr. Villarojo 1,000 cases of glasswares of the
class at the price stipulated in the contract. Such delivery was to be made during the first
Monday of the months of February and March 2005. In this case, no further demand or
notice by Mr. Villarojo on Mr. Montecillo was necessary because
a. Time is of the essence of the contract
b. The obligation expressly so provides
c. The demand would be useless
d. Law so provides
166
. It is not a source of liability which we entitle the injured party to damages:
a. Culpa Aquiliana or negligence commited in the performance of a spontaneous act
b. MORA or delay
c. Dishonesty, malice or bad faith in the performance of an existing valid obligation
d. Contravention of the tenor of obligation
167
. Unless the law or the stipulation of the parties requires another standard care, the obligation
to give a thing carries with it the obligation to take care of it with
a. Extraordinary diligence
b. Ordinary diligence
c. Diligence of a good father of a family
d. Special diligence
168
. If the law or contract does not state the diligence which is to observed in the performance of
an obligation, the standard of care required is
a. Ordinary diligence
b. Extra-ordinary diligence
c. Due diligence of a father of a good family
d. Proper diligence of a good father of a family
169
. In the three of the following instances, the office manager may still be held liable for fortuitous
event. Which is the exception?
a. If he assumed the management in bad faith.
b. If he undertakes risky operations like the owner was accustomed to do
c. If he is manifestly unit to carry on the management
d. If he has preferred his own interest to that of the owner.
170
. All are characteristics of a fortuitous event except:
a. It is beyond ones control
b. It is impossible to foresee
c. It is impossible to avoid if can be foreseen
d. It will directly or indirectly make the performance of obligation very difficult.
171
. In three of the following cases the debtor is still liable even if the thing is lost due to fortuitous
event. Which is the exception?
a. When debtor is already in delay
b. When obligation’s nature requires assumption of risk
c. When the object involves a delimited generic
d. When debtor is partly negligent.
172
. On June 5, 2005, Jose Dizon of Angeles City is obliged to give to Ruben Samia, Jose Dizon’s
red crew cab. There was no delivery until June 15, 2005 when the garage of the red crew
cab collapsed due to heavy ash and sand spewed by Mt. Pinatubo and the red crew cab was
totally destroyed. After the crew cab was destroyed and lost, is Jose Dizon still liable?

1 - 32
REGULATORY FRAMEWORK FOR BUSINESS TRANSACTIONS R. LOPEZ

a. No, even if Jose Dizon was already in default, he could plead impossibility of
performance.
b. Yes, the obligation to deliver the crew cab is changed to pay the equivalent value
because Jose Dizon is in legal delay.
c. No, because there was no demand by Ruben Samia to deliver the crew cab and the
specific object was lost due to fortuitous event. The obligation is extinguished.
d. Yes, because the contract is perfected.
173
. OE is obliged to give OR his 1982 Mitsubishi, 4-door sedan Lancer car with plate number
NBA123 on September 30, 2005. On October 10, 2005, OE did not yet deliver the car which
was totally destroyed by an earthquake on such date. Is OE still liable?
a. No, the obligation is extinguished. The specific thing was lost due to fortuitous event and
no demand to deliver was made by OR.
b. Yes, OE is in legal delay. OR can claim damages.
c. No, even OE is already in default and can plead impossibility of performance
d. Yes, OR can instead demand for another car of equivalent value from OE.
174
. Mr. ABC is obliged to give Mr. XYZ his only car on July 15,2005. Mr. ABC did not deliver the
car on July 15, 2005. On July 20, 2005 an earthquake destroyed the building where the car
was in a garage and the car was destroyed. Is Mr. ABC still liable?
a. No, considering that no demand to deliver was made by Mr. XYZ and the specific thing
was lost due to fortuitous event, the obligation is extinguished.
b. No. The obligation is extinguished, even if the debtor is already in default, because the
debtor can plead impossibility of performance.
c. Yes. Mr. ABC is already in legal delay, thus the obligation to deliver the lost specific thing
is converted into monetary claim for damages.
d. Yes. The creditor can instead demand for a substitute of equivalent value from the debtor.
175
. A is obliged to give B his only car on September 1, 2005. On the said date, A did not deliver.
On September 2, 2005, an earthquake completely destroyed the car. Is A still liable?
a. No, considering that no demand to deliver was made by Mr. XYZ and the specific thing
was lost due to fortuitous event, the obligation is extinguished.
b. No. The obligation is extinguished, even if the debtor is already in default, because the
debtor can plead impossibility of performance.
c. Yes. Mr. ABC is already in legal delay, thus the obligation to deliver the lost specific thing
is converted into monetary claim for damages.
d. Yes. The creditor can instead demand for a substitute of equivalent value from the debtor.
176
. X is under obligation to deliver his Toyota car to Y. However, before delivery Z destroys the
car. Which one of the following is not correct?
a. X’s obligation to give his car to Y is extinguished.
b. X is allowed to recover from Z.
c. Y has the right to bring action against Z.
d. X is not obliged to give Y an equivalent value of the car.
177
. Demand is not necessary in the following cases except:
a. When time is of the essence
b. When obligation so provides
c. When the law so provides
d. In contract of sale with separate dates for payment and delivery.
178
. P.A.U sold to R.D.E the former’s car on April 1, 2005 for P300,000. P.A.U. promised to deliver
the car on April 15, 2005 but R.D.E. did not make any promise when to pay.
a. P.A.U. on April 15, 2005 can demand payment from R.D.E. because the obligation is not
subject any condition.
b. R.D.E. will be obliged to pay only at or after delivery.
c. P.A.U. cannot demand payment until he delivers the car.
d. R.D.E. can compel P.A.U. to deliver the car before April 15, 2005 upon payment of the
selling price.
179
. The buyer of a thing has the right to the fruits of the thing from the time the
a. Thing bought is delivered.

1 - 33
REGULATORY FRAMEWORK FOR BUSINESS TRANSACTIONS R. LOPEZ

b. Sale is perfected.
c. Obligation to deliver the thing bought arises.
d. Fruits are delivered.
180
. Y sold his horse to Z for P50,000. No payment has yet been made and the sales document
does not provide the date of delivery. Before delivery and payment, the horse gave birth to a
baby horse.
a. Z is entitled to the baby horse which was born after the perfection of the contract.
b. Y is entitled to the fruit (baby horse) as Z has not paid the price yet.
c. Y is entitled to the fruit (baby horse) because it was born before his obligation to deliver
the horse.
d. Z should pay additional amount for the baby horse to be entitled to it.
181
. A borrower who uses the thing for a purpose different from that intended, delays its return,
receives the thing under appraisal, lends it to a third person, or saves his property instead of
the thing borrowed shall be liable even in case of fortuitous event, because
a. The nature of the obigation requires the assumption of risk.
b. The parties have expressly stipulated such liability.
c. The law expressly so provides
d. Answer not given.
182
. Every obligation whose performance does not depend upon a future or uncertain event, or
upon a past event unknown to the parties, is demandable at once. This refers to
a. Divisible and indivisible obligations.
b. Joint and solidary obligations.
c. Obligation with a period.
d. Pure obligations.
183
. If the obligation of the debtor is “I will pay you my debt after I have arrived from abroad”, this
is
a. Unenforceable
b. With a period
c. Void
d. Conditional
184
. If the obligor binds himself to perform his obligation as soon as “he shall have obtained a
loan” from a certain bank, this obligation is
a. With a term
b. Conditional
c. Suspensive
d. Resolutory
185
. A promissory note signed by Martiniano and dated March 15, 2005 is worded as follows: “I
promise to pay Juanita the sum of Fifty Thousand Pesos (P50,000) provided that if she
should fall in the October, 2005 CPA Examination, she shall return me the said amount.” The
above notes give rise to an obligation with
a. Suspensive condition
b. Casual condition
c. Resolutory condition
d. Mixed condition
186
. The passage of time extinguishes the obligation
a. Rescission
b. Prescription
c. Fulfillment of resolutory condition
d. Arrival of resolutory condition
187
. The happening of the time extinguishes the obligation
a. Prescription
b. Resolutory condition
c. Divisible obligation
d. Potestative condition

1 - 34
REGULATORY FRAMEWORK FOR BUSINESS TRANSACTIONS R. LOPEZ

188
. “A sells to B his lot and house in the city if A decides to transfer and live in the countryside” is
an example of
a. Mixed condition
b. Potestative condition
c. Casual condition
d. Resolutory condition
189
. A condition which, if imposed on an obligation will be disregarded and will therefore make the
obligation immediately demandable.
a. If Juan commits suicide.
b. If Juan passes the bar examination
c. If Juan kills Pedro.
d. If Juan will not rise from the dead.
190
. When the thing deteriorates pending the fulfillment of the suspensive condition without the
fault of the debtor, the impairment is to be borne:
a. By the party who caused the deterioration
b. Party by the debtor and partly by the creditor
c. By the debtor
d. By the creditor
191
. When the debtor binds himself to pay when his means permit him to do so, this kind of
obligor is called
a. A conditional obligation
b. A solidary obligation
c. An indivisible obligation
d. An obligation with a period.
192
. The obligation is demandable on the date of the obligation and shall continue to be in force
up to the arrival of the day certain:
a. Resolutory period.
b. Suspensive period.
c. Indefinite period.
d. Legal period.
193
. The following cases, the court may fix the period except one:
a. When the period is dependent on the sole will of the debtor
b. When the debtor binds himself to pay when his means permit him
c. When period is intended by the parties but not fixed
d. When the debtor loses the benefit of the period.
194
. In three (3) of the following cases advanced payment by the debtor is not recoverable. Which
is the exception?
a. The obligation was not yet due and demandable but the debtor believed it was already
due and demandable.
b. The payment is only for interest and credited to the proper period.
c. The advanced payments were made by both parties reciprocally.
d. The debtor was not aware of the period.
195
. In which of these cases, is advanced payment recoverable by the debtor
a. If the debtor believed that the obligation was already due and demandable
b. If the debtor was not unaware of the period.
c. If the obligation is reciprocal and both parties advanced payment
d. If the payment is only of interest credited for the proper period.
196
. Whenever a period is designated in an obligation, it is presumed to have been established
for the benefit of
a. Both the creditor and the debtor
b. The creditor
c. The debtor
d. The third party.

1 - 35
REGULATORY FRAMEWORK FOR BUSINESS TRANSACTIONS R. LOPEZ

197
. The debtor loses the benefit of the period, and his obligations become demandable when
a. After contracting the obligation, the creditor suspects that the debtor is becoming
insolvent.
b. The debtor attempts to abscond.
c. The guarantees as promised and delivered by the debtor are not acceptable to the
creditor.
d. Demand by the creditor would be useless.
198
. X, Y and Z are solidary liable to A for P30,000 which mature on July 1 1993. On May 1, 1993,
X paid A for the whole amount of the debt. If on December 1, 1993, X will be reimbursed by
Y, the latter will be liable for
a. P10,000 with interest from July 1, 1993 to December 1, 1993
b. P10,000 without interest
c. P10,000 with interest from May 1, 1993 to July 1, 1993
d. P10,000 with interest from May 1, 1993 to December 1, 1993
199
. A, B and C secured a loan from X. The promissory note which evidences the obligation
states: “I promise to pay” and signed by A, B and C. The obligation is
a. Joint
b. Solidary
c. Divisible
d. Indivisible
200
. A, B and C borrowed P24,000 from Y and Z and signed a promissory note dated January 15,
2005 and due within six (6) months. How much can Y collect from A?
a. P12,0000
b. P4,000
c. P8,000
d. P24,000
201
. This is the promissory note: “We promise to pay A, B and C the sum of ONE HUNDRED
EIGHTY THOUSAND (P180,000) PESOS within 60 days. Signed: X, Y and Z.
a. X is obliged to pay A P20,000
b. X is obliged to pay A P60,000
c. X is obliged to pay A P180,000
d. X is obliged to pay A, B and C P180,000.
202
. Carliot, Alfredo and Francis owed in solidum P15,000 to Ricardo as evidenced by a
promissory note due on September 30, 2005. The note prescribed on September 30, 2005.
On October 10, 2005, Carlito paid Ricardo. In this case, Carlito is
a. Entitled to collect P5,000 each from Alfredo and Francis
b. Not entitled to reimbursement from his co-debtors for the shares of the latter.
c. Entitled to recover from Ricardo
d. Answer not given.
203
. Payment of the obligation by a solidary debtor shall not entitle him to reimbursement from his
co-debtors.
a. If such payment was made before the obligation is due.
b. If such payment was made after the obligation has prescribed or become illegal.
c. If such payment was made in compliance with the demand made to him by all the
creditors
d. If such payment was made after the obligation has become due and demandable and
notice of payment was made only to him.
204
. A, B and C arte solidary debtors of X in the amount of P12,000. X then made a demand from
A but was able to collect P8,000 only because X was remitting A’s share of P2,000. How
much can A collect from B and C?
a. P12,000
b. P8,000 each
c. P6,000 each
d. P4,000 each.

1 - 36
REGULATORY FRAMEWORK FOR BUSINESS TRANSACTIONS R. LOPEZ

205
. A, B and C solidarily owe X and Y P30,000. X remitted the entire obligation in favor of A. The
effect is
a. The obligation is not extinguished until A collects from B and C.
b. The obligation is not yet extinguished until Y is paid by X and Y’s share of the credit.
c. A cannot recover from B and C because remission in his favor extend to the benefit of B
and C.
d. A can recover from B and C their respective share of the debt.
206
. A and B are joint creditors mancomunados of X and Y, joint debtors, for a total sum of
P9,000. A owes 1/3 of the credit; B owns 2/3 of it. But X owes 2/5 of the debts; Y owes 3/5 of
the debt. In this case
a. B can only collect from X, P5,400 and from Y, P3,600
b. A can only collect from X, P4,500 and from Y, P4,500
c. A can only collect from X, P3,600 and from Y, P5,400
d. A can only collect from X, P9,000 and from Y, P0.
207
. X, Y and Z solidarily owe A, B, C, D, joint creditors to the amount of P30,000. How much can
A collect from X?
a. P7,500 only
b. P20,000, A in turn has to give P5,000 each to B, C and D
c. P10,000, A in turn has to give P2,500 each to B, C and D
d. P5,000 only.
208
. X and Y are solidary debtors of A, B and C, joint creditors to the amount of P30,000. How
much can B collect from X?
a. B can collect P30,000 from X. B in turn has to give A and C P10,000 each.
b. B can collect P10,000 from X
c. B can collect P15,000 from X.
d. B can collect P30,000 from X. X in turn can recover from Y the amount of P15,000.
209
. Carlo and Cleo are solidary debtors of Corrina, Luis, Edward, and Abelito, joint creditors in
the amount of P20,000. How much can Corrina collect from Carlo?
a. Corrina could collect P2,500 from Carlo.
b. Corrina could collect P10,000 from Carlo. Corrina is then obliged to give P2,500 each to
Luis, Edward and Abelito.
c. Corrina could collect P5,000 from Carlo.
d. Corrina could collect the whole P20,000 from Carlo, but will in turn given P5,000 each to
Luis, Edward and Abelito.
210
. X and Y are solidary debtors of A, B, C and D, joint creditors to the amount of P8,000. How
much can A collect from X?
a. A could recover P4,000 from X. A, in turn has to give to B, C , & D, P1,000 each.
b. A could recover P2,000 only from X.
c. A could recover P1,000 only from X.
d. A could recover P8,000 from X, A in turn has to give B, C & D P2,000 each.
211
. A and B are solidary debtors of W, X, Y and Z, joint creditors to the amount of P2,000. How
much can W collect from debtor A?
a. W could recover P2,000 from A. W, in turn, has to give P500 each to X, Y and Z.
b. W could recover P1,000 from A. W, in turn has to give P250 each to X, Y and Z.
c. W could recover P500 only from A.
d. W could recover P250 only from A.
212
. X, Y and Z joint debtors owe P18,000 to A, B and C solidary creditors. How much can B
collect from X?
a. P3,000
b. P18,000
c. P9,000
d. P6,000
213
. A and B are solidary debtors of X, Y and Z, joint creditors to the amount of P15,000. How
much can Z collect from B?

1 - 37
REGULATORY FRAMEWORK FOR BUSINESS TRANSACTIONS R. LOPEZ

a. Z could recover P7,500 from B


b. Z could recover P5,000 from B
c. Z could recover P15,000 from B. Z in turn has to give X and Y P5,000 each
d. Z could recover P15,000 from B. B in turn can collect from A the amount of P7,500.
214
. A, B and C secured a loan from D. The promissory note which evidence the obligation states:
“I promise to pay D or order P10,000 payable on demand.” Signed A, B, C. The obligation is
a. Solidary
b. Divisible
c. Indivisible
d. Joint.
215
. Diaz is under obligation to give Cruz, at Diaz option, a car, a pick-up or a van.

1st Statement: If the car and the pick-up were destroyed by Diaz own fault and later Diaz
communicated to Cruz to deliver the van, but got lost by a fortuitous event, Diaz should not
be held liable.
2nd Statement: If the car and the pick-up were destroyed by a fortuitous event and later the
van was lost by Diaz’ fault, Diaz should be held liable
a. Correct, wrong
b. Wrong, correct
c. Correct, correct
d. Wrong, wrong
216
. The creditor shall have a right to indemnity for damages when, through the fault of the
debtor, all the things which are alternatively the object of the obligation have been lost or
compliance of the obligation has become impossible. The indemnity shall be fixed taking as a
basis the value of the
a. Least expensive thing
b. Most expensive thing
c. Last thing which disappeared
d. First thing which disappeared.
217
. An obligation where only the prestation has been agreed upon but the debtor may render
another in substitution is
a. Conjoint obligation
b. Facultative obligation
c. Simple obligation
d. Alternative obligation
218
. It refers to a joint obligation
a. One in which each debtor is liable for the entire obligation, and each creditor is entitled to
demand the whole obligation.
b. One in which either one of the parties is indispensable and the other is not necessary
c. One in which the obligation of one is a resolutory condition of the obligation of the other,
the non-fulfillment of which entitles the other party to rescind the contract.
d. One in which each of the debtors is liable only for a proportionate part of the debt and
each creditor is entitled only for a proportionate part of the credit.
219
. A solidary obligation is one in which each of the debtors is liable for the entire obligation or
debt, and each of the creditors is entitled to the entire credit. Obligations shall also be
considered solidary under the three following exceptions. Which does not belong to the
exception?
a. When solidarity is expressly stipulated in the obligation.
b. When the prestation is indivisible and there are two or more debtors and creditors.
c. When the law expressly provides solidarity.
d. When solidarity is required from the nature of the obligation/
220
. In a joint obligation, A, B and C are debtors of joint creditors D, E and F in the amount of
P180,000. A’s obligation is
a. Pay D P60,000
b. Pay D, E and F, P180,000

1 - 38
REGULATORY FRAMEWORK FOR BUSINESS TRANSACTIONS R. LOPEZ

c. Pay D P120,000
d. Pay D P20,000
221
. Obligation which is susceptible of partial performance based on the purpose of the obligation
a. Solidary
b. Divisible
c. Joint
d. None of the above
222
. Bertfulo and Claudio promise to deliver a particular car valued at P100,000 to Manuela on or
before September 15, 2005. September 15, 2005 came and upon demand by Manuela for
delivery from Bertfulo and Claudio, Bertfulo was willing to deliver but Claudio refused to
deliver. In the case at bar
a. An action for specific performance will lie against both Bertfulo and Claudio
b. Both Bertfulo and Claudio shall be liable for P50,000 each with damages.
c. Bertfulo shall be liable for P50,000 without damages and Claudio shall be liable for
P50,000 and damages
d. Answer not given.
223
. A, B and C solidarily bound themselves to deliver to X a Honda Motorcycle. The obligation
was not fulfilled through the fault of A. Thereupon, X filed an action in court against C and the
court awarded P36,000 to X. Which of the following situations is valid?
a. If C pays X the P36,000, C can collect from A P26,000 and B P12,000. Later, B can ask
for reimbursement from A P12,000.
b. X has to collect P12,000 each from A, B and C to satisfy the court’s award of P36,000.
c. If C pays X the P36,000, C can collect from B P10,000 and from A P16,000.
d. X cannot collect the whole amount of P36,000 from C.
224
. A person is entitled to adequate compensation only for such pecuniary loss suffered by him
as he has duty proved, except as provided by law or by stipulation. Such payment is referred
to as
a. Nominal damages
b. Compensatory damages
c. Liquidated damages
d. Monetary damages
225
. X obligated himself to pay Y the amount of P30,000 30 days after May 31, 2005 plus a
penalty of P3,000 if he fails to pay the obligation on due date. After demand for payment by
Y, offered to pay on July 30, 2005. Y can demand from X
a. P30,000 plus P3,000 plus legal interest
b. P30,000 plus legal interest
c. P30,000 plus P3,000
d. P30,000 plus P3,000 plus legal interest plus damages.
226
. The following shall produce the effect of payment of debts
a. Delivery of check
b. Tender of Central Bank notes
c. Delivery of promissory note
d. Answer not given.
227
. A owes B P10,000. With consent of both, C pays B P5,000. Now B and C are the creditors of
A to the amount of P5,000 each. Suppose A has only P5,000. Which is correct?
a. B and C should divide the P5,000 equally.
b. C should be preferred.
c. A may choose whom to pay.
d. B should be preferred.
228
. X owes Y P40,000. With the consent of both parties, Z pays Y P20,000 which makes Y and Z
creditors of X to the amount of P20,000 each. X becomes bankrupt and has only P20,000
assets left.
a. Z should get the P20,000
b. Y and Z should divide the P20,000 equally.

1 - 39
REGULATORY FRAMEWORK FOR BUSINESS TRANSACTIONS R. LOPEZ

c. X may choose whom to pay.


d. Y should get the P20,000.
229
. On October 4, 2005, A was indebted to B for P50,000 for a 20-day period. A proposed to B
that X will pay A’s debt, and that A will be free from all liabilities. B and X agree to the
proposal. On October 25, 2005, when B tries to collect from X, he finds that X is insolvent. At
the time of delegation, X was insolvent but this was not known to A. the insolvency is not of
public knowledge. So B sues A on the ground that it was A who made the proposal and that A
really guaranteed X’s solvency. Decide.
a. A is liable because he is presumed to have guaranteed X solvency.
b. A is not liable, because A does not know the insolvency of X at the time of delegation and
neither was the insolvency of public knowledge.
c. A is liable because he did not exercise due diligence in determining the insolvency of X.
d. A is liable because X agree to the proposal to make himself solidarily liable for the
obligation.
230
. This special form of payment is the conveyance of the ownership of a thing as an accepted
equivalent of performance.
a. Dation in payment or dacion en pago.
b. Assignment of property or cession en pago
c. Application of payment
d. Tender of payment and consignation.
231
. This is a mode of extinguishing obligation as when building is transferred to a creditor in
satisfaction of a debt in money and is governed by the law on sales.
a. Renunciation
b. Payment by cession
c. Consignation
d. Dacion en pago
232
. Dacion en pago as distinguished from sale.
a. The object is always existing and specific.
b. There is a greater degree of freedom in fixing the price.
c. There is no pre-existing obligation.
d. The cause is the price.
233
. Which of the following is not a special form of payment?
a. Cession
b. Application of payment
c. Consignation
d. Dacion in payment
234
. The P1,000 bills issued by the Central Bank and in circulation are considered
a. Checks
b. Bills of exchange
c. Legal tender
d. Promissory notes
235
. To have the effect of payment, debts in money should be made
a. In currency which is legal tender in the Philippines
b. By the delivery of promissory notes payable to order.
c. By the delivery of checks or bills of exchange
d. By all of the above.
236
. One of the following statements is not true:
a. A creditor is not bound to accept a check in satisfaction of his demand, because a check
even if good when offered, does not meet the requirements of legal tender.
b. The obligation of a debtor who agreed to pay in dollars in a foreign bill of exchange, shall
be discharged in Philippine currency measured at the prevailing rate of exchange at the
time the obligation is incurred.

1 - 40
REGULATORY FRAMEWORK FOR BUSINESS TRANSACTIONS R. LOPEZ

c. The Philippine peso bills hen attempted to be exported, as when carried in excess of that
allowed by the CB regulation, may be deemed to have been taken out of domestic
circulation as legal tender and thus, treated as commodity.
d. The purchasing power or value of money or currency depends upon, can come into
being, can be created or brought about by a law enacted by the legislative department of
the Government.
237
. In the absence of an agreement to the contrary what shall be the basis of payment of an
obligation in case there should supervene, an extraordinary inflation or deflation of the
currency stipulated?
a. The value of the currency at the time of the establishment of the obligation
b. The value of the currency at the due date of the obligation
c. The value of the currency shall be fixed by courts
d. The value of the currency shall be determined by an expert from the Central Bank
238
. The leviable properties of the debtor are transferred to the creditors to be sold and from the
proceeds, the creditors are paid:
a. Pledge
b. Cession
c. Dacion en pago
d. Chattel mortgage
239
. Assignment or abandonment of all the properties of the debtor for the benefit of his creditors
in order that the latter may sell the same and apply the proceeds thereof to the satisfaction of
his credits.
a. Remission
b. Expromission
c. Payment by cession
d. None of the above.
240
. It presupposes not only that the obligor is able, ready, and willing, but more so, in the act of
performing his obligation.
a. Promissory note
b. Tender of payment
c. Bill of exchange
d. Obligation to sell
241
. Extinguishment of debt may be made by the creditor either verbally or in writing.
a. Implied remission
b. Express remission
c. Partial payment
d. None of the above
242
. If the creditor to whom tender of payment has been made refuses without just cause to
accept it, the debtor shall be released from responsibility by
a. Assignment of property
b. Consignation of the thing or sum due
c. Adjudication or dacion en pago
d. Answer not given.
243
. Consignation is a mode of payment which extinguishes an obligation. Which of the following
is not a requisite for consignation?
a. Actual consignation with the proper judicial authorities.
b. Prior tender has not been made.
c. Existence of a valid debt
d. There must be prior notice of consignation to persons interested in the fulfillment of the
obligation.
244
. A thing is considered lost when it
a. Perishes
b. Goes out of commerce
c. Stolen or robbed

1 - 41
REGULATORY FRAMEWORK FOR BUSINESS TRANSACTIONS R. LOPEZ

d. All of them
245
. A thing is not deemed lost when it
a. Perishes
b. Disappears in such a way its existence is unknown or it cannot be recovered
c. Goes out of commerce
d. Deteriorates
246
. When the subject matter of a contract is lost through a fortuitous event, who is liable?
a. The debtor
b. The creditor
c. None of them
d. Both creditor and debtor
247
. One is not a requisite needed in order that obligation shall be extinguished by loss or
destruction of the thing due:
a. When the thing is lost without the fault of debtor.
b. When the thing lost is generic
c. When the thing is lost before the debtor has incurred in delay
d. When the thing lost is specific.
248
. An act or liberty whereby a credit condones the obligation of the debtor.
a. Confusion
b. Compensation
c. Remission
d. Novation
249
. A owes solidary creditors XYZ P10,000. There is remission of the debts when
a. X borrows P10,000 from A.
b. X waives the whole obligation of P10,000 to A.
c. Z makes a will giving the P10,000 debt to A as legacy
d. Y tells A that instead of paying P10,000, A shall just deliver a ring to Y.
250
. ABC are solidary creditors of X for P10,000. A makes a will giving the P10,000 debt to X as
legacy. The obligation is extinguished by
a. Confusion of debt
b. Novation of debt
c. Remission of debt
d. Compensation of debt
251
. When the characters of the creditor and the debtor are merged in one and the same person,
there is extinguishment of the obligation by
a. Compensation
b. Merger of rights
c. Novation
d. Remission
252
. A executes a promissory note in favor of B and the promissory note is negotiated by B and
subsequently is indorsed in favor of A. the obligation to pay the promissory note is thereby
extinguished because there is
a. Confusion or merger
b. Novation
c. Remission
d. Condonation
253
. When two persons in their own right are reciprocally creditors and debtors of each other and
extinguishes both debts to the concurrent amount, what takes place is known as
a. Compensation
b. Novation
c. Confusion or merger
d. Remission

1 - 42
REGULATORY FRAMEWORK FOR BUSINESS TRANSACTIONS R. LOPEZ

254
. A owes B P2,000 demandable and due on September 10, 2005. B, on the other hand, owes
A P2,000 demandable and due on or before September 30, 2005. If B claims compensation
on September 10, 2005, can A rightfully oppose?
a. No, B, who was giving the benefit of the term, may claim compensation because he could
then choose to pay his debt on September 10, 2005 which is actually “on or before
September 30, 1984”.
b. Yes, A can properly oppose because for compensation to take place, mutual consent of
both parties is necessary.
c. Yes, A can properly oppose and if B still refused to accept his payment made on
September 10, 2005, A can deposit his payment in court.
d. None of the above.
255
. Which one of the following is not an element of legal compensation?
a. Debts to be compensated are due and demandable
b. There is controversy or adverse claim over any debts to be compensated
c. There are two or more debts of the same kind
d. There are two persons who are creditors and debtors of each other.
256
. Which of the examples given below can legal compensation take place?
a. A owes B P1,000 due on June 30, 2005. B owes A P1,000 due on June 30, 2005 but C
has filed an adverse claim against A.
b. A owes B P1,000 payable October 31, 2005. B owes A P1,000 due on October 31, 2005.
c. A owes B P1,000 with C as his guarantor. B owes C P1,000.
d. A owes B P1,000 payable May 31, 2005. B to deliver to A 2 piculs of sugar worth P1,000
on May 31, 2005.
257
. Jesus owes Maria P1,000 on June 30, 2005. Maria owes Jesus P600 due on June 20, 2005.
Maria owes Jesus P400 due on June 30, 205.

On June 25, 2005 Maria cannot pay Jose P1,000 so she assigns to Jose her credit of P1,000
against Jesus without the knowledge of Jesus. On July 2, 2005 Jose tries to collect from
Jesus the P1, 000. How much can Jose compel Jesus to pay?
a. Zero
b. P1,000
c. P600
d. P400
258
. Change of person or objects
a. Confusion
b. Novation
c. Solutio indebiti
d. None of the above
259
. A obliged himself to give B a car if B places among the top ten in CPA Board Exam.
Subsequently, they agreed that A will give B the car if B merely passes the CPA Board. This
is an example of
a. Mixed novation
b. Real novation
c. Implied novation
d. Personal novation
260
. The distinction between conventional subrogation and assignment is that in conventional
subrogation
a. It is a mere cession of right
b. An obligation is extinguished and another appears
c. The same obligation, without being extinguished, is transferred to another.
d. Answer not given.

END OF QUIZZER

1 - 43
REGULATORY FRAMEWORK FOR BUSINESS TRANSACTIONS R. LOPEZ

1 - 44
1
. Letter “A” is the correct answer.
Article 3, Section 18 (2) of the 1987 Philippine Constitution provides, that: “’No involuntary servitude in any form shall
exist except as a punishment for a crime whereof the party shall have been duly convicted”.
Involuntary servitude includes enforced, compulsory service of one to another. It has been applied to any service or
labor, which is not free, no matter under what form such service, may have been rendered, a stipulation compelling the
debtor to remain as a servant in the house and in the service of her creditor so long as she had not paid her debt is
considered an involuntary servitude. An involuntary servitude is unlawful; it violates the constitutional rights of the debtor.
Forced labor is similar to slavery.
Letter “A” is incorrect because, contrary to good custom is not a precise answer because there is a law which
specifically prohibit involuntary servitude.
Letter “B” is incorrect because it is not only against a public policy but contrary to a specific provision of the Philippine
Constitution, thus it is contrary to law and morality.
Letter “D” is incorrect, because the rule mentioned is still subject to the limitation that the obligation must be lawful.
2
. Letter “A” is the correct answer.
An offer is a proposal to make a contract. If an offer is made through an agent its constitutes a proposal by a principal
which is deemed accepted only from the time acceptance is communicated to him (Art. 1322 of the New Civil Code).
Letter “B” is incorrect because a business advertisement of things for sales are not definite offers, but mere invitation to
make an offer (Art. 1325, supra).
Letter “C” is incorrect because advertisement for bidders are simply invitation to make proposal and the advertiser is not
bound to accept the highest or lowest bidder, unless the contrary appears (Art. 1326, supra).
Letter “D” is incorrect because politician refers to an imperfect promise or offer hence defective.
3
. Letter “B” is the correct answer.
When one of the contracting parties is compelled by a reasonable and well-grounded fear of an imminent and grave evil
upon his person or property, or upon the person or property of his spouse, descendants or ascendants, to give his
consent there is intimidation (Art. 1325 (2) of the New Civil Code).
Letter “A” is incorrect because there is violence when in order to wrest consent, serious or irresistible force is employed
(Art. 1335 [1] supra).
Letter “C’ is incorrect because undue influence happen when a person take improper advantage of his power over the
will of another thus, depriving the latter of a reasonable freedom of choice (Art. 1337, supra).
Letter “D” is incorrect because reverential fear (or fear out of respect/ displeasing another) does not vitiate consent.
4
. Letter “C” is the correct answer.
Simulation of contract may be absolute or relative. It is absolute when the parties do not intend to be bound at all and
relative when the contracting parties conceal their true agreement (Art. 1345, New Civil Code).
Letter “A” is incorrect because when the parties do not intend to be bound at all the simulation of contract is absolute.
The basic characteristic of absolutely simulated contract is the fact that the apparent contract is not really desired or
intended to produce legal effects in any way alter druidical situation of the parties.
Letter “B” is incorrect because the contract is not void. The parties are still bound by their real agreement, provided that
it does not prejudice a third person and it is not intended for any purposes contrary to law, morals, and good customs,
public order or public policy (Article 1346, supra).
Letter “D” is incorrect because contract is likewise void.
5
. Letter “A” is the correct answer.
It is specifically provided that if mistake, fraud, inequitable conduct, or accident has prevented a meeting of the minds of
the parties, the proper remedy is not reformation of the instruments but annulment of the contract (Article 1359 [2], New
Civil Code).
Letter “B” is incorrect because, if mutual mistake of the parties causes the failure of the instrument to disclose their real
agreement, the proper remedy is reformation of the instrument (Article 1361, supra).
Letter “C” is incorrect because, if one party was mistaken and the other knew or believed that the instrument did not
state their real agreement, but concealed that fact from the former, the instrument may be reformed (Article 1363,
supra).
Letter “D” is incorrect since there is no need to annul because contract is still valid provided their true agreement is
lawful and no third person is prejudiced.
6
. Letter “C” is the correct answer.
By express provision, voidable contracts which could be subject to ratification are those where one of the parties is
incapable of giving his or her consent, or those that do not comply with the Statute of Fraud, or where both parties are
incapable of giving their consent (Article 1403, New Civil Code). Ratification extinguishes the action to annul a voidable
contract (Article 1392, New Civil Code).
Letter “A” is incorrect because novation extinguishes a contract when there is a change in the object or principal
conditions, substitution in the person of the debtor or subrogation of third person in the rights of the creditor (Article
1291, supra). It does not extinguish the action to annul voidable contract.
Letter “B” is incorrect because rescission is a subsidiary remedy when the party suffering damage has no other legal
means to obtain reparation in a rescissible contract (Article 1383, supra), it does not extinguish action to annul voidable
contract. The action for annulment of a voidable contract is extinguished only by two modes, namely: 1) prescription
and/or; 2) ratification.
Letter “D” is incorrect because it is the extinguishment of obligation by the passage of time.
7
. Letter “B” is the correct answer.
Traditio longa manu is delivery by mere consent or agreement; it happens if the movable sold cannot yet be transferred
to the possession of the buyer at the time of the sale (Article 1499, New Civil Code).
Letter “A” is incorrect because in ratio and symbolica, delivery is with regard to movable property, which may also be
made by the delivery of the keys of the place or depository where it is stored or kept (Article 1498 [2], supra). In delivery
by traditio brevi manu, the buyer had already the possession of the object even before the purchase (Article 1499,
supra).
Letter “D” is incorrect because there is no need for actual delivery since the seller keeps possession of the property.
8
. Letter “A” is the correct answer.
It is a settled principle that when the law establishes the obligation and the act or condition upon which it is based is
nothing more than a factor for determining the moment when it becomes demandable, then the law itself is the source of
the obligation.
The Workmen’s Compensation Act is a special law enacted to protect the rights of the employees. The obligation of the
employer to pay the death and funeral expenses of his employee’s death while in the course of employment is
specifically provided in the Act.
Letter “B” is incorrect because the basis of the obligation is not the contract but the law which specifically impose
obligation upon the employer. And even if the parties agree to the contrary, such agreement is void being contrary to law.
Letter “C” is incorrect because there is no quasi contract but by express provision of the law.
Letter “D” is incorrect because quasi-delicts refer to extra contractual obligations arising from ones negligence resulting
to injury to another but no existing contract between them.
9
. Letter “C” is the correct answer.
Article 1173 (2) of the Civil Codes provides that: “If the law of contract does not state the diligence which is to be
observed in the performance, that which is expected of a good father of a family shall be required”.
Letter “A” is incorrect because extra ordinary diligence unless specifically provides is required in transportation law.
Letter “B” is incorrect because ordinary diligence is not precise for the law specifically states diligence of a good father of
the family.
Letter “D” is incorrect because of the same explanation in letter B.
10
. Letter “A” is the correct answer.
As a rule, those obliged to deliver or to do something incur in delay from the time the obligee judicially or extra judicially
demands from them the fulfillment of the obligation. However, the demand by the creditor shall not be necessary in order
that delay may exit: a) when the obligation or the law expressly so declares; b) when from the nature and circumstances
of the obligation it appears that the designation of the time when the thing is to be delivered or the service is to be
rendered was a controlling motive for the establishment of the contract and; c) when demand would be useless, as when
the obligor has rendered it beyond his power to perform (Article 1169, New Civil Code).
While the contract does not expressly provide the date when to deliver the 1,000 cases of glasswares, it is fact that
delivery was to be made during the months of February and March for the said reunions before the end of the said
months. The parties agree the period of delivery for a purpose, which was the controlling motive for the establishment of
contract. Thus, demand is not necessary.
Letter “B” is incorrect because the contract only provides for the object which is the promise to deliver the 1,000 cases of
glass wares of the class and the stipulated price, but not as to the exact date of delivery without demand.
Letter “C” is incorrect because the demand is useless only when the obligor is rendered it beyond his power to perform,
as when he promise to deliver it to two persons not having the same interest.
Letter “D” is incorrect because it is not applicable here. An example of which is date for payment of tax.
11
. Letter “C” is the correct answer.
It is provided that except in cases expressly specified by the law, or when it is otherwise declared by stipulation or when
the nature of the obligation requires the assumption of risk, no person shall be responsible for those events which could
not be foreseen, or which, though foreseen, were inevitable (Art. 1174, New Civil Code).
As a general rule, the obligor is not liable for damage or delay caused by fortuitous event. However, by express
provision, if the obligor delays, or has promised to deliver the same thing to two or more persons who do not have the
same interest, he shall be responsible for any fortuitous event until he has effected the delivery (Article 1165, supra).
Letter “A” is incorrect because the obligor is liable by express provision of the law not by he nature of the obligation.
Letter “B” is incorrect because even if the parties do not expressly stipulated such liability, the obligor is guilty of bad
faith.
Letter “D” is incorrect because it is not applicable in the situation since no crime/delict is mentioned in the problem.
12
. Letter “B” is the correct answer.
As expressly provided, when the debtor binds himself to pay when his means permit him to do so, the obligation shall be
deemed to be one with a period (Article 1180, New Civil Code).
Letter “A” is incorrect because the condition is not dependents on the debtor’s sole will but by express provision the
obligation is one with a period.
Letter “C” is incorrect because the obligation is not void.
Letter “D” is incorrect because it is a condition the happening of which will extinguish the obligation hence immediately
demandable.
13
. Letter “A” is the correct answer.
Compensation shall take place when two persons, in their own right, are creditors and debtors of each other (Article
1278, New Civil Code).
Letter “B” is incorrect because novation extinguishes a contract when there is a change in the object or principal
condition, substitution in the person of the debtor or subrogation of third person in the rights of the creditor (Article 1291,
supra).
Letter “C” is incorrect because confusion or merger happens when the characters of the creditor and debtor merged in
the same person (Article 1275, supra).
Letter “D” is incorrect because remission which is essentially gratuitous requires the acceptance of the obligor, subject to
the rules governing inofficious donations (Article 1270, supra).
14
. Letter “B” is the correct answer.
If the creditor to whom tender of payment has been made refuses without just cause to accept it, the debtor shall be
released from responsibility by the consignation of the thing or sum due (Article 1256 [1], New Civil Code).
Letter “A” is incorrect because in assignment of property, when the debtor cede or assign his property to his creditors in
payment of his debts and unless there is stipulation to the contrary, shall only relapse the debtor from responsibility for
the net proceeds of the thing assigned (Article 1255, supra).l
Letter “C” is incorrect because dacion en pago is alienation of property to the creditor in satisfaction of the debt in money
(Article 1245, supra).
Letter “D” is incorrect because cession or assignment in favor of creditors is another term for letter (A).
15
. Letter “A” is the correct answer.
There is confusion because the characters of creditor and debtor are merged in the same person. When the promissory
note initially endorsed by A in payment for his debt to B was subsequently endorsed in his favor, Mr. A becomes a debtor
a creditor at the same time. The instrument he issued in answer for his obligation was indorsed to him in payment for the
obligation of another person.
Letter “B” is incorrect because there is novation only when novation extinguishes a contract when there is a change in
the object or principal conditions, substitution in the person of the debtor or subrogation of the third person in the rights
of the creditor (Article 1291, supra). Here there is no substitution of debtor or subrogation of creditor because the
creditor and debtor are merged in one and the same person.
Letter “C” is incorrect because remission which is essentially gratuitous requires the acceptance of the obligor, is subject
to the rules governing inofficious donations (Article 1270, supra).
Letter “D” is incorrect because condonation which is essentially gratuitous requires the acceptance of the obligor, subject
to the rules governing in officious donations (Article 1270, supra).
16
. Letter “C” is the correct answer.
Compensation shall take place when two persons, in their own right, are creditors and debtors of each other (Article
1278, New Civil Code).
Letter “A” is incorrect because compensation shall not be proper when one of the debts arises from a depositum or from
the obligations of a depositary or of a bailee in commodatum (Article 1287, supra).
Letter “B” is incorrect because, neither shall there compensation if one of the debts consist in civil liability arising from
penal offense (Article 1288, supra).
Letter “D” is incorrect because compensation takes place only if they will agree.
17
. Letter “B” is the correct answer.
The distinction between subrogation and assignment of credit are as follows:
a) In subrogation, the debtor’s consent is necessary while in assignment it is not;
b) Subrogation creates a new obligation and at the same time extinguishes the old one while in assignment, the same
right exist but there is only a change in the person of the creditor; and
c) Defects in the old obligation may be cured by subrogation but defects in the old obligation are not remedied by
assignment of credit.
18
. Letter “C” is the correct answer.
By a chattel mortgage, personal property is recorded in the Chattel Mortgage Register as a security for the performance
of the obligation. If the movable, instead of being recorded, is delivered to the creditor or a third person, the contract is
pledge and not chattel mortgage (Article 2140, New Civil Code).
19
. Letter “C” is the correct answer.
The fulfillment of the obligation is done through traditio longa manu, which is a delivery by mere consent or agreement
and it happens even if the movable sold cannot yet be transferred to the possession of the buyer at the time of the sale
(Article 1499, New Civil Code). Although there is no actual delivery yet, the contract of sale is consummated by the
agreement that for payment of the sum of 5,000.00 Mr. Abalos could harvest at any time he likes on any tree Mr. Tala-od
pointed.
Letter “A” is incorrect because in traditio brevi manu, the buyer had already the possession of the object even before the
purchase (Article 1499 of the NCC), here Mr. Abalos is not yet in possession of the mangoes.
Letter “B” is incorrect because in traditio symbolica, delivery is done the delivery of the keys or the place or depository
where it is stored or kept (Article 1498, [2]).
Letter “D” is incorrect because there is no need for actual delivery since the seller keeps possession of the property.

20
. Letter “C” is the correct answer.
In conditional obligations, the acquisition of rights, as well as the extinguishments or loss of those already acquired, shall
depend upon the happening of the event Whig constitutes the condition (Article 1181, New Civil Code).
When Martiniano signed the promissory note his obligation to pay Juanita the sum of Fifty Thousand Pesos (P50,
000.00) is immediately demandable, and will be terminated upon the happening of the event that is, if she fails the CPA
Board Exams. The obligation is immediately effective and demandable, without prejudice to the happening of the
resolutory condition.
Letter “A” is incorrect because in suspensive condition, it is the happening of the event which gives rise to the obligation.
The obligation of Mariano to pay is not dependent of the passing of Juanita in the Board, as worded in the promissory
note it is immediately demandable but subject to recovery if she failed in the exams.
Letter “B” is incorrect because the happening of the event is not dependent upon chance or the will of a stranger.
Letter “D” is incorrect because it is another term for suspensive condition.
21
. Letter “C” is the correct answer.
A joint indivisible obligation gives rise to indemnity for damages from the time anyone of the debtors does not comply
with his undertaking. The debtors who may have been ready to fulfill their promise shall not contribute to the indemnity
beyond the corresponding portion of the price of the thing or of the value of the service in which the obligation consists
(Article 1224, New Civil Code).
A promise to deliver a particular car is one of joint indivisible obligation and failure to deliver the same gives rise to
indemnity for damages.
Letter “A” is incorrect because Bertulfo was willing to deliver the car. He shall not be liable for damages but only for the
portion of the price of the car that is P50, 000.00.
Letter “B” is incorrect because only Claudio who refused to deliver without justifiable caused is the only one liable to pay
damages.
Letter “D” is incorrect because it is a remedy if obligation is to deliver generic thing.
22
. Letter “B” is the correct answer.
Payment by a solidary debtor shall not entitle him to reimbursement from his co-debtors if such payment is made after
the obligation has prescribed or become illegal (Article 1218, New Civil Code).
Letter “A” is incorrect because when Carlito paid Ricardo in October 10, 2005, the obligation prescribed in September
30, 2005, thus he is not entitled to collect P5,000.00 each from his co-debtors.
Letter “C” is incorrect because Carlito cannot recover from Ricardo. As a rule, when a right to sue upon a civil obligation
has lapsed by extinctive prescription, the obligor who voluntarily performs the contract cannot recover what he has
delivered or the value of the service he has rendered (Article 1424, supra).
Letter “D” is incorrect as explained in letter B.
23
. Letter “B” is the correct answer.
A contract is voidable when the consent of one of the contracting parties is vitiated by mistake, violence, intimidation,
undue influence or fraud (Article 1390).
Letter “A” is incorrect because the sale is voidable and not void. Violence, intimidation and undue influence vitiated the
consent of Eduardo. He was kidnapped and subjected to torture if not for which, he will not sell his land to Cecilio.
Letter “C” is incorrect because the contract is defective and by express provision it is not valid by voidable.
Letter “D” is incorrect because rescission is a remedy if there is lesion or insufficiency in cause or consideration.
24
. Letter “C” is the correct answer.
Since Corrina is a joint creditor she can collect only the share which corresponds to her credit, in this case P5, 000.00.
Letter “A” is incorrect because Carlo and Cleo are solidary debtors and Corrina can demand entire compliance of the
obligation but only as to her share which is P5, 000.00 and not P2, 500.00.
Letter “B” is incorrect because Corrina cannot collect P10, 000.00 from Carlo, as joint creditor hiss hare from the credit is
only P5, 000.00.
Letter “D” is incorrect because Corrina can neither collect the entire obligation of P20, 000.00, because their relationship
is governed by the rules of joint obligation.
25
. Letter “C” is the correct answer.
As a rule, the vendee is bound accept delivery and to pay the price of the thing sold at the time and place stipulated in
the contract. If the time and place should not have been stipulated, the payment must be made at the time and place of
the delivery of the thing sold (Article 1582, New Civil Code).
Letter “A” is incorrect because P.A.U. only promise to deliver the car on April 15, 1992, there is no actual delivery yet,
thus he can demand payment on such date.
Letter “B” is incorrect because in absence of any contract as to when payment must be made, payment must be made at
the time and place of the delivery of the thing sold.
Letter “D” is incorrect because the answer is uncalled for, the question is when payment should be made not when
R.D.E., the buyer could demand delivery of the car.
26
. Letter “B” is the correct answer.
By express provision of law, when the consent is vitiated by mistake, violence, intimidation, undue influence or fraud, the
contract is voidable (Article 1390, New Civil Code).
Letter “A” is incorrect because when the contract is undertaken in fraud of creditors, the contract could be subject to
rescission (Article 1381 [3], supra).
Letter “C” is incorrect because when the object is outside the commerce of man, the contract is void (Article 1409 [4],
supra).
Letter “D” is incorrect because when both parties are incapable of giving consent, the contract is unenforceable (Article
1403 [3], supra).

27
. Letter “B” is the correct answer.
A contract is at the conception period when it is at a stage starting from negotiation till the moment just before the
agreement of the parties.
Letter “A” is incorrect because a contract is perfected at the moment there is a meeting of the minds upon the thing
which is the object of the contract and upon the price (Article 1475, New Civil Code).
Letter “C” is incorrect because when the parties come to an agreement, there is already the meeting of the minds.
Letter “D” is incorrect for the same reasons previously given.

28
. Letter “A” is the correct answer.
Before an obligation could be validly demanded the same must be due and demandable, otherwise the demand is
premature. If Mr. Debtor dies before December 31, 1991which is the due date, Mr. Creditor cannot collect from Mr.
Debtor until after it is due and demandable and the debt will be paid out of the estate of the deceased not against the
heirs of Mr. Debtor.
Letter “B” is incorrect because the debt is not yet due and thus cannot be demanded until December 31, 1991 arrived.
Letter “C” is incorrect because the two legitimate emancipated children of Mr. Creditor, if he dies can recover from Mr.
Debtor. As a rule, subject to the law, all rights acquired in virtue of an obligation are transmissible, if there has been no
stipulation to the contrary (Article 1178, New Civil Code).
Letter “D” is incorrect because the recovery will be against the estate of Mr. Debtor not from his heirs.
29
. Letter “A” is the correct answer.
When only one prestation has been agreed upon, but the obligor may render another in substitution, the obligation is
called facultative (Article 1206, New Civil Code).
Letter “B” is incorrect because simple obligation is an obligation with only one object constituting the prestation.
Letter “C” is incorrect because alternative obligation is those where although several prestation are due the complete
performance of one will extinguish the obligation.
Letter “D” is incorrect because conjoint obligation indicates multiplicity of obligation arising from plurality of debtors or
creditors.
30
. Letter “D” is the correct answer.
A is oblige to pay D the sum of P20, 000.00 (same with E and F). The share of A with the debt which is P180, 000.00 is
divided among his co-debtors amounting to P60, 000.00 each into as many equal shares as there are creditors.
Letter “A” is incorrect because by the express provision, that if from the law, or the nature or the wording of the
obligations to which the proceeding article refers the contrary does not appear, the credit or debt shall be presumed to
be divided into as many equal shares as there are creditors or debtors, the credits or debts being considered distinct
from one another, subject to the Rule of Court governing the multiplicity of suit (Article 1208, New Civil Code). The share
of D in so far as A is concerned is P20, 000.00 not P60, 000.00.
Letter “B” is incorrect because, A is not obliged to pay P180, 000.00 because this joint obligation not solidary.
Letter “C” is incorrect for same reason previously given.
31
. Letter “C’ is the correct answer.
The obligor in an obligation to deliver or to do something incurs delay only from the time the obligee judicially or extra-
judicially demands fulfillment of the obligation (Article 1169, New Civil Code). Although the obligation of Jose Dizon to
deliver the red crew car is on June 5, 1991, he is not in delay yet when it was collapsed and totally destroyed due to Mt.
Pinatubo in June 15, 1991 because no demand to deliver was made by Ruben Samia, the obligee.
Letter “A” is incorrect because if Jose Dizon is already in default he could not plead impossibility of performance. As a
rule, if the obligor delays, he shall be responsible for any fortuitous event until he has affected the delivery (Art. 1165,
supra).
Letter “B” is incorrect, there is no legal delay because there is no demand which will place the obligor in delay.
Letter “D” is incorrect because while there is a perfected contract there is no delay which will make the obligor liable for
damages.
32
. Letter “A” is the correct answer.
One of the requisites of a valid contract is that the consent, which constitutes an offer, must be certain and the
acceptance is absolute. A qualified acceptance constitutes a counter-offer (Article 1319 of the Civil Code). Here, by the
terms of the letter, there is only an offer to sell a 500 sq. m. lot for P300, 000.00. By increasing the amount to P350,
000.00 there is a qualified acceptance that he agrees to sell the lot but in a higher price, there is no meeting of the minds
yet. Thus, Mr. Aguas cannot compel Mr. Santos to accept the P300, 000.00 and make him sign and execute a Deed of
Sale.
Letter “B” is incorrect because Mr. A is not estopped because his second letter constitutes qualified acceptance and not
absolute.
Letter “C” is incorrect because since there is no offer certain and absolute acceptance there is no meeting of minds yet.
Letter “D” is incorrect because, in absence of a consent, there is no perfected contract.
33
. Letter “D” is the correct answer.
A contract of partnership having a capital of three thousand pesos or more, in money or property, shall appear in public
instrument and recorded with SEC. However, failure to comply thereto shall not affect the liability of the partnership and
the members thereof to third person (Article 1772, New Civil Code).
Letter “A” is incorrect because a contract asking for impossible service is void, as provided, impossible condition in those
contrary to good customs or public policy and those prohibited by law shall annul the obligation which depends upon
them (Article 1183, supra).
Letter “B” is incorrect because when a sale of a piece of land or any interest therein is through an agent, the authority of
the latter shall be in writing, otherwise the sale shall be void (Article 1874, supra).
Letter “C’ is incorrect because a contract of partnership is void, whenever immovable property is contributed thereto, if
an inventory of said property is not made, signed by the parties, and attached to the public instrument (Article 1773,
supra).
34
. Letter “C” is the correct answer.
A stipulation forbidding the owner from alienating the immovable mortgage shall be void (Article 2130, New Civil Code).
Letter “A” is incorrect because, by empress provision of the law, the agreement not to sell is void and this is contrary to
public policy.
Letter “B” is incorrect because a stipulation wherein the mortgagor is required to get the consent of the mortgagee as
without legal basis. The mortgage encumbers but it does not end ownership, the mortgagor remains the owner of the
property, it could be foreclose (McCullough v. Veloso, 46 Phil. 1).
Letter “D” is incorrect because the owner could alienate his property even during the existence of the mortgage.
35
. Letter “B” is the correct answer.
When the performance of the obligation depends if the obligor will obtain a loan from a certain bank, the obligation is
conditional. In conditional obligation, the acquisition of rights, as well as the extinguishments or loss of those already
acquired, shall depend upon the happening of the event which constitutes the condition (Article 1181, New Civil Code).
Obligor will pay if the bank will grant him loan, otherwise no payment will be made.
Letter “A” is incorrect because an obligation is one with a term if the debtor binds himself to pay when his means permit
him to do so (Article 1180, supra). The obligation to pay is dependent not only from the availability of means of the
obligor but the loan he will obtain from the bank.
Letter “C” is incorrect because, in suspensive condition, it is the happening of the event which gives rise to the
obligation.
Letter “D” is incorrect because, in resolutory condition, the obligation is immediately effective and demandable, without
prejudice to the happening of the resolutory condition.
36
. Letter “D’ is the correct answer.
Insane or demented persons are among those who cannot give a valid consent to a contract (Art. 1327, New Civil
Code). But contract entered into during a lucid interval are valid (Art. 1328, supra). While Mr. Esguerra was confined in
the mental hospital in 1982 he was released under the guardianship of his wife, thereafter and became a mining
inspector in 1984. it was during this period that he sold some his mining claims. Presumption arises that he is of sound
mind if not in lucid interval when he is selling his mining shares. As a rule, he who claims insanity must judicially prove
the same, in case of doubt presumption favors sanity, thus the sale is valid.
Letter “A” is incorrect because, the sale is not illegal, Mr. Esguerra at the time he enter the contract has legal capacity,
thus he could validly give consent to the contract.
Letter “B” is incorrect because, the cause object or purpose not being contrary to law, morals, good customs, public
order or public policy, it is not void. Letter “C” is incorrect because the contract is not voidable, because Mr. Esguerra
when he enters into contract is capable of giving a valid consent.
37
. Letter “C” is the correct answer.
By express provision, contracts agreed to in a state of drunkenness or during hypnotic spell are voidable (Article 1328,
New Civil Code). The effect of the intoxicating liquor must obscure or enfeeble the mental faculties of one of the
contracting parties. Hypnotic spell should render the mental faculties incapable of proper functioning. Being on such
state of mind, such parties incapable of giving consent to a contract.
Letter “A” is incorrect because the contract is voidable not void if one of the contracting parties is in state of drunkenness
or under a hypnotic spell when entering into a contract.
Letter “B” is incorrect because the contract is not valid because of the defect in the giving of consent of one of the
contracting parties. Letter “D” is incorrect because the contract is not necessarily legal or illegal it is rather a defective
contract.
38
. Letter “C” is the correct answer.
The P1,000.00 bills issued by the Central Bank and in circulation is considered legal tender.
Letter “A” is incorrect because checks are bill of exchange drawn on a bank and payable on demand (Section 185,
Negotiable Instruments Law). This is not legal tender, it produce the effect of payment only when encashed.
Letter “B” is incorrect because a bill of exchange is unconditional order in writing addressed by one person to another,
signed by the person giving it, requiring the person to whom it is addressed to pay on demand or at a fixed or
determinable future time a sum in certain in money to order or to bearer (Section 126, supra). It is negotiable instrument
not legal tender.
Letter “D” is incorrect because a promissory note is an unconditional promise in writing made by one person to another,
signed by maker, engaging to pay on demand or at a fixed or determinable future time, a sum certain in money or to
bearer (Section 184, supra). It is not a legal tender but another form of negotiable instrument.
39
. Letter “D” is the correct answer.
It is the primary objective of the Bangko Sentral to maintain the price stability conducive to a balanced and sustainable
growth if the economy. It promotes and maintains monetary stability and the convertibility of peso (Section 3, The New
Central Bank Act).
Letter “A” is incorrect because a creditor is not bound to accept a check in satisfaction of his demands. As provided,
checks representing demand deposits do not have legal tender power and their acceptance in the payment of debts,
both public and private, is at the option of the creditor (Section 60, supra).
Letter “B” is incorrect because in case of extraordinary inflation or defilation of the currency stipulated should supervene,
the value of the currency at the time of ht e establishment of the obligation shall be the basis of payment, unless there is
agreement to the contrary (Article 1250, New Civil Code).
Letter “C” is incorrect.
40
. Letter “B” is the correct answer.
The elements of the obligation are: juridical tie, active and passive subject and object or prestation.
Letter “A” is incorrect because a maker or drawer is one of the requirements of a negotiable instrument, in addition it
must be in writing and signed by the maker or drawer (Section 1[a], Negotiable Instruments Law).
Letter “C” is incorrect because for the same reason previously given and as provided when the instrument is addressed
to a drawee he must be named or otherwise indicated therein with a reasonable certainty (Section 1 [e], supra).
Letter “D” is incorrect because money is also one of the requirements of a negotiable instrument. As stated a negotiable
instrument must contain an unconditional promise or order to pay a sum certain in money (Section 1 [b], supra).
41
. Letter “D” is the correct answer.
The contract is valid, reluctance and failure to exercise good sense and judgment do not vitiate the consent of the
contracting party.
Letter “A” is incorrect because the contract is not void because the cause, object or purpose is lawful and within the
commerce of man.
Letter “B” is incorrect because, there is no vitiated consent, reluctance and failure to exercise good sense and judgment
do not constitute mistake unless it is a gross mistake.
Letter “C” is incorrect because there is no showing that the contracting parties are both incapable giving their consent to
render the contract unenforceable.
42
. Letter “B” is the correct answer.
Simulation of contract may be absolute or relative. There is absolute simulation of contract when the parties do not
intend to be bound at all; the latter, when the parties conceal their agreement (Article 1345, New Civil Code). An
absolute simulated or fictitious contract is void.
Letter “A” is incorrect because there is an existing contracts but the parties either by mistake or fraud do not intend to be
bound at all or conceal their true agreement.
Letter “C” is incorrect because a contract through relatively simulated when it does not prejudice a third person and is
not intended for any purpose contrary to law, morals, good customs, public order or public policy binds the parties to
their real agreement (Article 1346, supra).
Letter “D” is incorrect by express provision of the law.
43
. Letter “B” is the correct answer.
By express provision of the law, neither shall rescission take place when the things, which are the object of the contract,
are legally in the possession of third person who did not act in good faith (Article 1385, new Civil code).
Letter “A” is incorrect because though it contemplate the spirit of the law that rescission can take place when the object
of the contract is in the possession of the third party acting in bad faith, but by process of elimination.
Letter “B” is incorrect because more and exactly the same as worded by the law.
Letter “C” is incorrect because it can be carried out only when he who demands rescission can return whatever he may
be obliged to restore (supra). Partial performance and partial rescission is not allowed.
Letter “D” is incorrect because, by express provision, rescission creates the obligation to return the things which were
the object of the contract, together with their fruits, and the price with its interest (supra). If the seller cannot return the
installment paid to him by the buyer, no rescission can take place.
44
. Letter “D” is the correct answer.
If from the law, or the nature or the wording of the obligations to which the preceding article refers the contrary does not
appear, the credit or debt shall be presumed to be divided into as many equal shares as there are creditors or debtors,
the credits or debts being considered distinct from one another, subject to the Rules of Court governing the multiplicity of
suit (Article 1208, New Civil Code).
Letter “A” is incorrect because if each debtor is liable for the entire obligation, and each creditor is entitled to demand the
entire obligation, the obligation is solidary.
Letter “B” is incorrect by express provision of the law.
Letter “C’ is incorrect for lack of legal basis.
45
. Letter “B” is the correct answer.
Tender of payment presupposes performance of obligation.
Letter “A” is the correct answer, promissory note is just an unconditional promise in writing made by one person to
another to pay a sum certain in money or to bearer (Section 184, Negotiable Instruments Law).
Letter “C’ is incorrect because a bill of exchange is an unconditional order in writing requiring the person to whom it is
addressed to pay on demand or at fixed or at determinable future time a sum certain in money to order or to bearer
(Section 126, supra). Both promissory note and bill of exchange, although negotiable are not legal tender and do not
presuppose an act of performance of obligation.
Letter “D” is incorrect by express provision of the law.
46
. Letter “A’ is the correct answer.
It is the answer because it should be Culpa contractual instead of Culpa acquilliana or negligence in the performance of
a pre-existing obligation which is a source of liability for damages. Culpa aquiliana is a source of obligation (under quasi-
delict).
Letters B, C, and D are incorrect because they are sources of liability.
47
. Letter “B” is the correct answer.
The contract is voidable by specific provision of law those where the consent is vitiated by mistake, violence,
intimidation, undue influence or fraud are voidable contracts (Article 1390, New Civil Code).
Letter “A” is incorrect because, a merged contract cannot be a subject of rescission because it can carried only when he
who demands rescission can return whatever he may be obliged to restore (Article 1385, supra). A spouse cannot return
the other spouse to her family should the marriage contract is rescinded.
Letter “C” is incorrect because although the consent of the husband is vitiated by intimidation, it is not within the
contemplation of a void contract.
Letter “D” is incorrect because when both parties are incapable of giving consent, the contract is unenforceable (Article
1403 [3], supra). Here only the consent of one of the contracting parties is vitiated.
48
. Letter “D” is the correct answer.
The debtor shall loose every right to make use of the period when by his own acts he has impaired guaranties or
securities after their establishment, and when through a fortuitous events their disappear, unless he gives new ones
equally satisfactory (Article 1198 [3], New Civil Code).
Although O has until November 23, 1993 to pay for his debts, his rights to make use of such period was lost by the
instruction of the mortgaged house due to fire. And the fact that he has not given a new equally satisfactory security
makes the obligation immediately due and demandable.
49
. Letter “A” is the correct answer.
By express provision, the defense of illegality of contracts is not available to third person whose interests are not directly
affected (Article 1421, New Civil Code). A contract binds only the parties thereto and only them can raise the defense of
its illegality except when the rights and interests of such third person are directly affected.
Letter “B” is incorrect because void contracts cannot be ratified. Neither can the right to set up the defense of illegality be
waived (Article 1409, supra).
Letter “C” is incorrect because for the reason previously given.
Letter “D” is incorrect because the action or defense for the declaration of the inexistence of a contract does not
prescribe (Article 1410, supra).
50
. Letter “A” is the correct answer.
Compensation shall take place when two persons, in their own right are the creditors or debtors of each other (Article
1278, new Civil Code). In order that compensation may be proper, it is necessary: 1) that each one of the obligor be
bound principally, and that he be at the same time a principal creditor of the other; 2) that both debts consist in a sum of
money, or if the thing due are consumable, they be of the same kind and also of the same quality if the latter has been
stated; 3) that the two debts be due; 4) that they be liquidated and demandable; and 5) that over neither of them there
be ant retention or controversy, commenced by third persons and communicated in due time to the debtor (Article 1279,
New Civil Code). Jesus and Maria are creditor and debtor of each other in their own rights. By way of legal
compensation, their credits and debts will be offset when both debts become due and demandable which is on June 30,
1993. Thus, Jose will get nothing while Jesus is indebted to Maria, she is also indebted to Jesus.
Letter “B” is incorrect because Jose cannot compel to pay P1, 000.00 because the amount Jesus owed to Maria is
compensated by the debt of the latter against Jesus.
Letter “C” is incorrect because when Jose tries to collect on July 2, 1993, all debts are due and demandable, thus legal
compensation has already taken place.
Letter “D” is incorrect for the same reason previously stated.
51
. Letter “C” is the correct answer.
By the contract of agency binds himself to render some service or to do something in representation or on behalf or
another, with the consent or authority of the latter (Article 1868, New Civil Code).
Letter “A” is incorrect because the piece of work is governed by the law on sales and it involves manufacturing of goods
especially for the customers and upon his special order, and not of the general market (Article 1467, new Civil Code).
Letter “B” is incorrect because contract of service is different from contract of agency, an agent bids himself to render
some service or do something in behalf of another under color of authority by the latter.
Letter “D” is incorrect because by specific provision of law the contract being referred to here is a contract of agency.
52
. Letter “C” is the correct answer.
By the contract of sale one of the contracting parties obligates himself to transfer the ownership of and to deliver a
determinate thing, and the other to pay therefore a price certain in money or its equivalent (Article 1458, New Civil
Code).
Letter “A” is incorrect because in delivery of goods to a distributor by consignment, the seller preserves the right of
ownership notwithstanding the delivery of the goods until certain condition has been fulfilled.
Letter “B” is incorrect because in sale the buyer after delivery becomes the owner while in agency to sell, like a
consignment for sale, the agent who is supposed to sell does not become the owner, even if the property has already
been delivered to him.
Letter “D” is incorrect because when goods are delivered to an agent to be sold by him and he is not liable to the
manufacturer of the good the contract is one of contract of agency to sell. The agent does warrant unlike a seller as long
as he acts within his authority and in the name of the principal.
53
. Letter “A” is the correct answer.
The stipulation in the mortgage that the land covered thereby shall become the property of the mortgagee upon failure to
pay the debt within the period agreed upon, constitute pactum commisorium and is therefore null and void (Tan Chun Tic
vs. West Coast life, 54 Phil. 361).
Letter “B” is incorrect because in consolidation of ownership, the ownership is merged in one person.
Letter “C” is incorrect because conventional redemption takes place when the vendor reserves the right to repurchase
the thing sold, with the obligation to comply with the provision of Article 1616 and other stipulation which may have been
agreed upon (Article 1601, supra).
Letter “D” is incorrect because consignation releases the debtor from responsibility if the creditor to whom tender of
payment has been made refuses without just cause to accept it by depositing the thing due at the disposal of judicial
authority.
54
. Letter “A” is the correct answer.
By express provision of the law, the contract of pledge or mortgage may secure all kinds of obligation, be they pure or
subject to a suspensive or resolutory condition (Article 2091, New Civil Code).
Letter “B” is incorrect because the sale of the thing pledged shall extinguish the principal obligation, whether or not the
proceeds of the sale are equal to the amount of the principal obligation, interest and expenses in a proper case. If the
price of the sale is more than said amount, the debtor shall not be entitled to the excess, unless it is otherwise agreed. If
the price of the sale is less, neither shall the creditor be entitled to recover the deficiency, notwithstanding any stipulation
to the contrary (Article 2115, supra).
Letter “C” is incorrect because the creditor cannot appropriate the things given by way to pledge or mortgage, or dispose
of them. Any stipulation to the contrary is null and void (Article 2088).
Letter “D” is incorrect for the same reason stated in letter A.
55
. Letter “D” is the correct answer.
A condition that Juan will not rise from the dead is a condition not to do an impossible thing which under the law shall be
considered as not having been agreed upon, thus the obligation is immediately demandable (Article 1183 [2], New Civil
Code).
Letter “A” is incorrect because condition that contrary to good customs or public policy and those prohibited by law shall
annul the obligation which depends upon them (supra).
Letter “B” is incorrect because this is an obligation subject to suspensive condition, the acquisition of rights as well as
the extinguishments or loss of those already acquired shall depend upon the happening of the event which constitutes
the condition (Article 1181, New Civil Code).
Letter “C” is incorrect for the reason stated in letter A.
56
. Letter “B” is the correct answer.
The obligation subject to potestative condition, because it is dependent on the will of A if he decides to transfer and live
to the countryside. As provided, a condition, which if dependent upon chance or upon the will of the third person, the
obligation shall take effect in the conformity with the provision of the Civil Code (Article 1182).
Letter “A” is incorrect because mixed condition happens if the obligation is subject on the will of third person and upon
chance.
Letter “C” is incorrect because casual condition is when the obligation is dependent upon chance.
Letter “D” is incorrect because resolutory condition is one which is demandable at once without prejudice to the effect of
the happening of the event (Article 1179, supra).
57
. Letter “C” is the correct answer.
By express condition, compensation shall take place when two persons, in their own right are creditors and debtors of
each other (Article 1278, New Civil Code).
Letter “A” is incorrect because there is confusion when the characters of creditor and debtor are merged in the same
person.
Letter “B” is incorrect because reformation is a remedy when there have been a meeting of a minds of the parties to a
contract but their true intention is not expressed in the instrument purporting to embody the agreement, by reason of
mistake, fraud, inequitable conduct or accident (Article 1359, supra).
Letter “D” is incorrect because there is novation when there is a change in the object or principal conditions, substitution
in the person of the debtor or subrogation of third person in the rights of the creditor (Article 1291, supra).
58
. Letter “D” is the correct answer.
The rule is payment to a person which is incapacitated to administer his property shall be valid if he has kept the thing
delivered, or insofar as the payment has been beneficial to him (Article 1241, New Civil Code). The money which was
deposited to the bank, spent for food, clothing and other personal expenses and use for tour abroad, amounting to P70,
000.00 redounded to the benefit of the minor. Thus, these are valid payment which upon reaching the age of majority the
minor can be required to return them.
Letter “A” is incorrect because the P20, 000 which he lost in gambling in a loan of P10, 000 to Z who became insolvent
are not considered valid payment, thus the minor cannot be required to return them.
Letters “B” and “C” are incorrect for the reason previously given.

59
. Letter “D” is the correct answer.
The first statement is correct, a valid object is one of the requisite of a valid contract, absence of which will render the
contract void ab initio and it cannot be remedied by providing a cause or object as the case may be. By specific
provision, all things which are not outside the commerce of men, including future things, may be the objects of a
contract. All rights which are not in transmissible may also be the objects of contracts (Article 1347, New Civil Code).
The second statement is also correct although not verbatim. It has the same meaning with Art. 1230.
60
. Letter “D” is the correct answer.
By express provision, the contract of sale is perfected at the moment there is a meeting of minds upon the thing which is
the object of the contract and upon the price (Article 1475).
Letter “A” is incorrect because, negotiation is a qualified acceptance, which could constitute an offer and counter offer.
Letter “B” is incorrect because there is no meeting of minds yet between the parties as to the subject matter and cause
of the interest.
Letter “C” is incorrect because what consummates the sale is the delivery which involves transfer of ownership.
61
. Letter “C” is the correct answer.
Gratuitous is not among the characteristics of pledge and mortgage its object is delivered or recorded by way of
collateral, to secure the fulfillment of the principal obligation normally loan or monetary obligation.
Letter “A” is incorrect because as provided one of the essential requisites of the contracts of pledge and mortgage is that
they are constituted to secure the fulfillment of a principal obligation (Article 1285 [1], New Civil Code).
Letter “B” is incorrect because, as a rule it is necessary, in order to constitute the contract of pledge, that the thing
pledged be placed in the possession of the creditor, or of a third person by common agreement (Article 2093, supra).
Delivery of the object pledge and recording with the Registry of Property or Chattel Mortgage Registry is essential in
pledge and mortgage. However, if the thing is placed in possession of third person, it could be decided by common
agreement.
Letter “D” is incorrect because, the object of pledge and mortgage are given always for consideration, that is to secure
principal obligation.
62
. Letter “B” is the correct answer.
There is real novation because there is a change in principal condition that instead of placing among top ten in the CPA
Board Exams, merely passing the board exams will give rise to an obligation that A will give his car to b.
Letter “A” is incorrect because, the change does not involve modification in the object or principal condition and in the
persons of the creditor and debtor.
Letter “C’ is incorrect because, the novation is not implied but rather express.
Letter “D” is incorrect because there is no substitution in the person of the debtor neither is there subrogation in the
rights of the creditor.
63
. Letter “C’ is the correct answer.
By express condition, compensation shall take place when two persons, in their own right, are creditor and debtor of
each other (Article 1278, New Civil Code).
Letter “A” is incorrect because reformation is a remedy when there have been a meeting of the minds of the parties to a
contract but their true intention is not expressed in the instrument purporting to embody the agreement, by reason of
mistake, fraud, inequitable conduct or accident (Article 1359, supra). Letter “B” is incorrect because, there is no
novation when there is a change in the object or principal condition, substitution in the person of the debtor or
subrogation of third person in the rights of the creditor (Article 1291, supra). Letter “D” is incorrect because there is
confusion when the characters of creditor and debtor are merged in the same person (Article 1275, supra).
64
. Letter “D’ is the correct answer.
To make the contract voidable, fraud must be serious and should not have been employed by both contracting parties.
The action for annulment of contracts shall be extinguished when the thing which is the object thereof is lost through the
fraud or fault of the person who has the right to institute the proceedings (Article 1401, New Civil Code).
Letter “A” is incorrect because, if the fraud is incidental, the recourse is to seek damages not to annul the contract. Letter
“B” is incorrect because, if both parties are in pari delicto, the law will leave them as they are, as if both parties acted in
good faith.
Letter “C” is incorrect for the reason previously stated.
65
. Letter “B” is the correct answer.
Basic is the rule in the law on obligation and contract that generic object is never lost. In an obligation to deliver a
generic thing, the lost or destruction of anything of the same kind does not extinguish the obligation (Article 1263, New
Civil Code). And if the thing is indeterminate or generic, the creditor may ask that the obligation be complied with at the
expense of the debtor (Article 1165, New Civil Code).
Letters “A”, “C”, and “D” are incorrect because, for obvious reason, these are requisites in order that obligation is
extinguished by lost or destruction of the thing due.
66
. Letter “C’ is the correct answer.
In an obligation with the penal clause, the penalty shall substitute the indemnity for damages and the payment of interest
in case of noncompliance, if there is no stipulation to the contrary (Article 1226, New Civil Code). Since Y is in delay, he
is liable to pay the principal obligation plus the penalty of P3, 000.00.
Letter “A” is incorrect because, the penalty of P3, 000.00 shall substitute the payment of interest, thus he need not pay
the legal interest.
Letter “B” is incorrect for the reason previously stated.
Letter “D” is incorrect by specific provision of the law.
67
. Letter “D” is the correct answer.
The following requisites are essential to the contract of pledge and mortgage: 1) that they be constituted to secure
fulfillment of principal obligation; 2) that the pledgor or mortgagor be the absolute owner of the thing pledged or
mortgaged and; 3) that the persons constituting the pledge or mortgage have the free disposal of their property, and in
the absence thereof, that they be legally authorized for the purpose (Article 2085, New Civil Code). In addition it is
necessary in order to constitute the contract of pledge, that the thing pledged be placed in the possession of the creditor,
or of a third person by common agreement (Article 2093, supra).
Delivery of the thing subject of pledge what constitute the contract of pledge, it is not necessary that the same be
recorded in the office of the Register of Deeds to bind third person, as in the case of mortgage.

Letters “A”, “B”, and “C” are incorrect, being self-explanatory.

68
. Letter “B” is the correct answer.
In payment by cession, the debtors cede or assign his property to his creditors in payment of his debts. This cession,
unless there is stipulation to the contrary, shall only release the debtor from responsibility for the net proceeds of the
thing assigned (Article 1225, new Civil Code).
Letter “A” is incorrect because, in pledge, the property is transferred by the debtor to his creditor only to secure
fulfillment of principal obligation and not to be sold for payment of his obligation.
Letter “C” is incorrect because, in Dacion en pago, a property is alienated to the creditor in the satisfaction of debt in
money (Article 1245, supra). The property need not sold to apply its proceed in payment of debt.
Letter “D” is incorrect because chattel mortgage just like pledge is intended to the secure fulfillment of principal
obligation.

69
. Letter “D” is the correct answer.
Letter “D” is correct because under the Statute of Fraud if the sale of a land is made orally, the same is unenforceable.
As provided, the sale of real property or of an interest therein must be in writing (Article 1403 [e], New Civil Code), hence
still valid.
Letter “A” is incorrect because, although the contract of sale of a land is in writing the authority of B to act as an agent is
orally, rendering the contract of sale void.
Letter “B” is incorrect because when a sale of a piece of land or any interest therein in through an agent, the authority of
the latter shall be in writing; otherwise, the sale shall be void. (Article 1874, New Civil Code). Likewise, the sale of real
property or an interest therein must be in writing (to be enforceable) (Article 1403 [e], New civil Code).
Letter “C” is incorrect because, by express provision of law, when the sale of piece of property or any interest therein is
through an agent, the authority of the latter shall be in writing; otherwise the sale shall be void (ibid).
70
. Letter “A’ is the correct answer.
Obligation with a resolutory period take effect at once, but terminate upon arrival of the day certain. A day certain is
understood to be that which must necessary come, although it may not be known when (Article 1193, new Civil Code).
Letter “B” is incorrect because in an obligation subject to suspensive period it is the arrival of the period which gives rise
to the obligation.
Letter “C’ is incorrect because, in an obligation subject to indefinite period, it is the court that fix the duration thereof, if
from its nature and circumstances it can be inferred that a period was intended (Article 1197, supra).
Letter “D” is incorrect for the same reason previously stated.
71
. Letter “A” is the correct answer.
This is a solidary obligation in absence of any stipulation to be contrary. The answer will be different if the promissory
note uses “We” instead of “I”, the former is joint obligation while the latter is a solidary obligation. The obligations of A, B,
and C are taken collectively, that is why they used the word “I”.
Letter “B” is incorrect because the divisibility of the things refer to the object of the obligations in which there is only one
debtor and one creditor
Letter “C” is incorrect because the indivisibility of the obligation does not necessarily give rise to solidarity. Nor does
solidarity of itself imply indivisibility (Article 1210, New Civil Code).
Letter “D” is incorrect because of the word “I” used in the problem as explained above.

72
. Letter “B” is the correct answer.
In order that compensation may be proper it is necessary: a) that each one of the obligors be bound principally and that
he be at the same time a principal creditor of the other; b) that both debts consist in a sum of money, or if the thing due
are consumable, they be of the same kind, and also of the sale quality the latter has been stated; c) that the two debts
be due; d) that they be liquidated and demandable; e) that over neither of them there be any retention or controversy,
commenced by third persons and communicated in due time to the debtor (Article 1279, New Civil Code).
From the above stated provision only when there is controversy or adverse claim over any debts to be compensated is
not an element of legal compensation. On the contrary over neither of them there be any retention or controversy,
commenced by third persons and communicated in due time to the debtor (ibid).

Letters A, C, and D are incorrect because, by specific provision of the law, the elements of a legal compensation is
exclusive.

73
. Letter “D” is the correct answer.
The share of each solidary creditors is P6, 000.00 by dividing the debt of P18, 000 into such numbers of creditors. It
bears stress, that while the governing relationship among the creditors is solidary, that of the debtors is joint. Thus the
creditor can demand entire compliance but only insofar as his share is concern not entire compliance of the prestation.
Letter “A” is incorrect because for lack of legal basis.
` Letter “B” is incorrect because the obligation of the debtors is joint and neither any of them can be compelled to pay the
entire obligation of P18, 000.
Letter “C” is incorrect for lack of legal basis.

74
. Letter “B” is the correct answer.
In an example wherein A owed B P1, 000 and B owes A the same amount both due and demandable on October 31,
1993, legal compensation set in. because each one of the obligors are bound principally as creditor and debtor of each
other, both debts consist in a sum of money and that they are both due and demandable on the same date.
Letter “A” is incorrect because it is necessary that over neither of boys debts there be any retention or controversy.
Letter “C” is incorrect because C is just a guarantor and not primarily liable as debtor.
Letter “D” is incorrect because both debts must consist in a sum of money and does not arise from any other kinds of
obligation.

75
. Letter “B” is the correct answer.
B as a joint creditor can collect only his share in the credit that is P10, 000.00. In anoints obligation, the share of the
creditor in the credit is to be divided into as many equal shares as there are creditors. B cannot demand entire
compliance although the debtors are solidarily bound.
Letter “A” is incorrect because B cannot collect the entire amount of P30, 000 because his share in the credit is divided
among his co-creditors. And he is entitled to collect only insofar as his share is concerned.
Letter “C” is incorrect for the lack of legal basis.
Letter “D” is incorrect, for the same reason previously stated.
76
. Letter “C” is the correct answer.
The following requisites are essential to the contract of pledge and mortgage: 1) that they be constituted to secure
fulfillment of principal obligation; 2) that the pledgor or mortgagor be the absolute owner of the thing pledged or
mortgaged and; 3) that the person constituting the pledge or mortgage have the free disposal of their property, and in
the absence thereof, that they be legally authorized for the purpose (Article 2085, New Civil Code).
Pledge and mortgage being an accessory contract exist by themselves. Their purpose is to secure fulfillment of the
principal obligation.
Letters “A”, “B” and “D” are incorrect because, these are true consequences of pledge and mortgage.
77
. Letter “A” is the correct answer.
Both cases are false. Contracts without case, or with unlawful cause, produce no effect whatever. The cause is unlawful
if it is contrary to law, morals, good customs, public order or public policy (Article 1352, New Civil Code). The contract
being one with unlawful cause, it does not produce any effect and A can recover whatever he has paid out of unlawful
contract.
The action for annulment of contract may be instituted by all who are thereby obliged principally or subsidiarily. However,
persons who are capable cannot allege the incapacity of those with whom they contracted; nor can those who exerted
intimidation, violence, or undue influence, or employed fraud, or caused mistake base their action upon these flaws of
the contract (Article 1379, supra). Thus, only B can institute action for annulment and not A who caused the intimidation.

78
. Letter “C” is the correct answer.
When a makes a will giving the P10,000 debt to X as legacy, there is remission. It is essentially gratuitous, and requires
the acceptance of the obligor and may be made expressly or impliedly. It shall be subject to the rules which govern
inofficious donation (Article 1270, New Civil Code).
Letter “A” is incorrect because there is confusion when the characters of creditor and debtor are merged in the same
person.
Letter “B” is incorrect because there is novation when there is a change in the object or principal conditions, substitution
in the person of the debtor or subrogation of third person in the rights of the creditor (Article 1291, supra).
Letter “D” is incorrect because there was no payment of debt since it was extinguished by inheritance or legacy.
79
. Letter “C’ is the correct answer.
Consignation alone is not a special form of payment but it releases the debtor from responsibility if the creditor to whom
tender of payment has been made refuses without just cause to accept it by depositing the thing due at the disposal of
judicial authority. Correct form should be tender of payment and consignation.
Letter “A” is incorrect because cession is a form of payment wherein the debtor cede or assign his property to his
creditors in payment of his debt. This cession, unless there is stipulation to the contrary, shall only release the debtor
from responsibility for the net proceeds of the thing assigned (Article 1255, New Civil Code).
Letter “B” is incorrect because application by payment is when the debtor has various debts of the same kind in favor of
one and the same creditor, he may declare at the time of making the payment, to which of them must be applied and is
also a special form of payment (Article 1252, supra).
Letter “D” is incorrect because in dacion en pago, a property is alienated to the creditor in satisfaction of debt in mone,
hence alos a special form of payment (Article 1245, supra).
80
. Letter “B” is the correct answer.
The contract is executed in fraud of creditors. And is not binding as between the contracting parties.
Letter “A” is incorrect because it is true that the contract is not valid for lack of due consideration because Mr. Gamboa
merely pretended that he executed a contract by virtue of which the debtor alienates his property by gratuitous title is
presumed to have been entered into in fraud of creditors, when the donor did not reserve sufficient property to pay all
debts contracted before the donation. Alienation by onerous title is also presumed fraudulent when made by persons
against whom some judgment has been rendered in any instance or some writ of attachment has been issued. The
decision or attachment need not refer to the property alienated, and need not have been obtained by the party seeking
the rescission. (Article 1387, New Civil Code).
Letter “D” is incorrect because by express provision Mr. Evangelista can rescind the contract, the contract is entered in
fraud of creditor thus it is a rescissible contract.
81
. Letter “C’ is the correct answer.
When Z, one of the creditors makes a will giving P10,000 debt to A as legacy, there is remission which is essentially
gratuitous. It is given without anything in exchange. It requires the acceptance of the obligor and may be made expressly
or impliedly. It shall be subject to the rules which govern inofficious donations. Express condonation shall, furthermore,
comply with the forms of donation (Article 1270, New Civil Code).
Letter “A” is incorrect because if X borrows P10, 000 from A, there will be a compensation as when two persons, in their
own right, are creditors and debtors of each other. Letter “B” is incorrect because partial waiver of obligation is not
remission of debts. Remission condones the entire obligation and not partially. Letter “D’ is incorrect because it is a form
of dacion en pago, as when a property is alienated to the creditor in satisfaction of debt in money (Article 1245, supra).
82
. Letter “C” is the correct answer.
The vendor is bound to transfer the ownership of and deliver, as well as warrant the thing, which is the object of the sale
(Article 1495, New Civil Code). Included in the obligation of the vendor is to execute necessary documents to affect the
transfer and delivery of the object.
Letter “A’ is incorrect because Mr. Cruz cannot occupy and use the land as a buyer in good faith because he is aware
that he has not yet registered the land in compliance with the land registration law. Letter “B” is incorrect because the
contract has already been consummated and therefore it can no longer be assailed as unenforceable. Letter “D” is
incorrect because Mr. Cruz should first demand Mr. Ayco to execute a deed of sale by way of exhaustion of remedies, in
this way he will not be saving his time and effort but also that of the court.
83
. Letter “B” is the correct answer.
The contract of mortgage need not be in writing, taking into consideration the provision of Article 1403 on Statute of
Fraud.
Letters “A”, “B”, and “D” are incorrect because the law provides that the following requisites are essential to the contracts
of pledge and mortgage: a) that they be constituted to secure the fulfillment of a principal obligation; b) that the pledgor
or mortgagor be the absolute owner of the thing pledged or mortgaged; c) that the person constituting the pledged or
mortgage have the free disposal of their property, and in the absence thereof, that they be legally authorized for the
purpose (Article 2085, new Civil Code).
84
. Letter “A” is the correct answer.
By express provision, a stipulation forbidding the owner from alienating the immovable mortgaged shall be void (Article
2130, New Civil Code).
Letter “B” is incorrect because the mortgaged credit may be alienated or assigned to a third person, in whole or in part,
with the formalities required by law (Article 2128, supra).
Letter “C” is incorrect because the creditor cannot appropriate the things given by way of pledge or mortgage, or dispose
of them. Any stipulation to the contrary is null and void (Article 2088, supra0.
Letter “D” is incorrect because it is indispensable, in order that a mortgage may be validly constituted, that the document
in which it appears be recorded in the Registry of Property. If the instrument is not recorded, the mortgage is
nevertheless binding between the parties (Article 2125, supra).
85
. Letter “B” is the correct answer.
There is confusion or merger happens when the characters of the creditor and debtor merged in the same person
(Article 1275, New Civil Code).
Letter “A” is incorrect because compensation arises when two persons, in their own right, are creditors and debtors of
each other (Article 1278, supra).
Letter “C” is incorrect because novation extinguishes a contract when there is a change in the object or principal
conditions, substitution in the person of the debtor or subrogation of third person in the rights of the creditor (Article
1291, supra).
Letter “D’ is incorrect because remission which is essentially gratuitous requires the acceptance of the obligor, subject to
the rules governing inofficious donations (Article 1270, supra).
86
. Letter “A” is the correct answer.
Simulation of contract may be absolute or relative. In a relative simulation of contract, there is an intentional deception
by producing the appearance of a contract which is different from the true agreement. But a contract through relatively
simulated when it does not prejudice a third person and is not intended for any purpose contrary to law, morals, good
customs, public order or public policy binds the parties to their real agreement (Article 1346, New Civil Code).
Letter “B” is incorrect because there is absolute simulation of contracts when the parties do not intend to be bound at all;
the latter, when the parties conceal their agreement (Article 1345, supra). An absolutely simulated or fictitious contract is
void.
Letter “C” is incorrect because in annulment of contract, the contract is terminated and will not produce effect
whatsoever.
Letter “D” is incorrect because misrepresentation when made by third person does not vitiate consent, unless such
misrepresentation has created substantial mistake and the same is mutual.
87
. Letter “C” is the correct answer.
Reformation of the instrument is made when, there having been a meeting of the minds of the parties to a contract, their
true intention is not expresses in the instrument purporting to embody the agreement, by reason of mistake, fraud,
inequitable conduct or accident, to the end that such true intention may be expressed (Article 1359, New Civil Code). A
contract of mortgage executed as a contract of sale can be reformed to express the true intention of the parties.
By express provision of the law, there shall be no reformation in the following cases: a) simple donations inter vivos
wherein no condition is imposed; b) wills; c) when the real agreement is void (Article 1366, supra). The reason is that the
testator can always change the will at his option without reforming it. Executing another will or testament is one way of
changing it.
88
. Letter “A” is the correct answer.
A as a joint creditor can collect only his share in the credit that is P7, 500.00. In a joint obligation, the share of the
creditor in the credit is to be divided into as many equal shares as there are creditors. He cannot demand entire
compliance although the debtors are solidarily bound.
Letter “B” is incorrect because A cannot collect the P20, 000.00 because his share in the credit is divided among his co-
creditors. And he is entitled to collect only insofar as his share is concerned.
Letter “C” is incorrect, for lack of legal service.
Letter “D” is incorrect for the same reason previously stated.
89
. Letter “A” is the correct answer.
If the thing has been lost or if the prestation has become impossible without the fault of the solidary debtors, the
obligation shall be extinguished. If there was fault on the part of any one of them, all shall be responsible to the creditor,
for the price and the payment of damages and interest, without prejudice to their action against the guilty or negligent
debtor (Article 1221, New Civil Code). Therefore, if C pays X the P36, 000.00, he can collect from A P24, 000.00 and
from B P12, 000.00. Later b can ask for reimbursement from A P12, 000.00. A who is the debtor at fault shall pay the
entire obligation and damages.
Letter “B” is incorrect because X need not collect P12, 000.00 each from the obligors since they are solidarily liable. He
can ask entire compliance with any of the obligors.
Letter “C” is incorrect because C need not share from the liability caused by A.
Letter “D” is incorrect because C is solidarily liable to A, thus he is bound to pay subject to reimbursement from the guilty
co-debtor.

90
. Letter “D” is the correct answer.
As provided, the contract of pledge and mortgage may secure all kinds of obligations, be they pure or subject to a
suspensive or resolutory condition (Article 2091, New Civil Code). A voidable or unenforceable contract until declared
void and annulled by the court are still valid and will produce legal effect same with the natural obligations, thus they can
be secured by constituting pledge.

Letter “A” is incorrect because it is necessary in order to constitute the contract of pledge, that the thing pledged be
placed in the possession of the creditor, or of a third person by common agreement (Article 2093, supra).
Letter “B” is incorrect because pledge is a real contract and it is perfected by the delivery of the thing pledged.
Letter “C” is incorrect because pledge is constituted to secure fulfillment of principal obligation, thus it is an accessory
contract.
91
. Letter “A” is the correct answer.
The stipulation in the mortgage that the land covered thereby shall become the property of the mortgagee upon failure to
pay the debt within the period agreed upon, constitute pactum commissorium and is therefore null and void (Tan Chun
Tic vs. West Coast Life, 54 Phil. 361).
The only right of the mortgages-creditor is to foreclose the mortgage if the mortgage is due and remains unpaid.
92
. Letter “C” is the correct answer.
This is a case of alternative obligation wherein the obligor is alternatively bound to deliver the different prestations but
performance of one is enough to extinguish the obligation. The right of choice belongs to the debtor, unless it has
expressly granted to the creditor (Articled 1200, New Civil Code). And the choice shall produce no effect except from the
time it has been communicated (Article 1201, supra). Although the car and the pick-up were destroyed by Diaz own
fault, liability will not attach against him because he has not yet communicated to Cruz what he opted to deliver at the
time they were lost. Even when he communicated his choice to deliver the van, the same was lost due to fortuitous
event.
If Diaz effected his choice before the loss, he will be held liable.
93
. Letter “B” is the correct answer.
There is fraud when, through insidious words or machinations of one of the contracting parties, the other is induced to
enter into a contract which, without them he would not have agreed to (Article 1338, new Civil Code).
Letter “A” is incorrect because there is undue influence when a person takes improper advantage of his power over the
will of another, depriving the latter of a reasonable freedom of choice. The following circumstances shall be considered:
the confidential, family, spiritual and other relations between the parties, of the fact that the persons alleged to have
been unduly influenced was suffering from mental weakness, or was ignorant or in financial distress (Article 1337,
supra).
Letter “C” is incorrect because mistake will invalidate consent if it refers to the substance of the thing which is the object
of the contract, or to those conditions which have been principally moved one or both parties to enter into the contract.
Letter “D” is incorrect because misrepresentation vitiates consent if it produces substantial mistake and the same is
mutual (Article 1342, supra).
94
. Letter “D” is the correct answer.
This is the case of novation, which consists in substitution of a new debtor in the place of the original one; it may be
made even without the knowledge or against the will of the latter, but not without the consent of the creditor (Article
1293, New Civil Code). If the substitution is without the knowledge or against the will of the debtor, the new debtor’s
insolvency or non-fulfillment of the obligation shall not give rise to any liability on the part of the original debtor (Article
1294, supra).
However, in the instant case, the substitution is with the consent of the original debtor who becomes bankrupt and has
only P20, 000.00 assets left. Such remaining asset of the original debtor will be applied to pay the balance of P20, 000
he owes to the creditor.
Letter “A” is incorrect because the new debtor cannot get the P20, 000.00 remaining asset of the original debtor
because the entire obligation of the latter has not yet been fully paid.
Letter “B” is incorrect because, Z cannot be subrogated to the rights of the creditor because the original obligation has
not yet been fully paid.
Letter “C” is incorrect because, X cannot choose whom to pay, the right of Y is preferred over Z.
95
. Letter “C” is the correct answer.
Both answers are wrong, there is no perfected contract. When Arrieta received the latter of Bascon completely accepting
his offer of sale in August 31, 1995, the offer has already been withdrawn in August 30, 1995, or the day before. Even if
he wrote the latter of acceptance in August 26, 1995, it was received after the offer was withdrawn. Acceptance made by
letter or telegram does not bind the offerer except from the time it came to his knowledge (Article 1319, New Civil Code).

96
. Letter “D” is the correct answer.
The obligation is subject to a condition that pay will be made provided the debtor will arrive from abroad. In conditional
obligations, the acquisition of rights, as well as the extinguishments or loss of those already acquired, shall depend upon
the happening of the event which constitutes the condition (Article 1181, New Civil Code).
Letter “A” is incorrect because the contract is not enforceable but valid subject to fulfillment of the condition.
Letter “B” is incorrect because the obligation is not subject to a period but the happening of a conditional event.
Letter “C” is incorrect because a conditional obligation is not void but the acquisition of rights is dependent on the
happening of the condition.
97
. Letter “B” is the correct answer.
When one of the contracting parties is incapable of giving consent, the contract is voidable and not enforceable (Article
1390, New Civil Code).
Letters A, C and D are incorrect because these are grounds for declaring a contract unenforceable by express provision
of the law.
98
. Letter “B” is the correct answer.
The commission agent cannot, without the express or implied consent of the principal, sell on credit. Should he do so,
the principal may demand from him payment in cash, but the commission agent shall be entitled to any interest or
benefit, which may result from such sale (Article 1905, New Civil Code).
Letter “A’ is incorrect because, A is not authorized to sell X’s TV set on credit, and if X will not ratify the sale he can
demand payment in cash for the agreed price which is P10, 000.00. Mariano is not entitled to commission.
Letter “C’ is incorrect because, the contract is not unenforceable, the contract is governed by Contracts of Agency and
not within the purview of unenforceable contracts.
Letter “D” is incorrect for the lack of legal basis.
99
. Letter “D” is the correct answer.
By express provision, a stipulation forbidding the owner from alienating the immovable mortgaged shall be void (Article
2130, New Civil Code). A property subject of mortgaged is never disposed of neither is there any transfer of ownership,
thus the mortgagor remains the owner and enjoys the rights and privileges afforded to an owner.
Letter “A” is incorrect because by specific provision of the law.
Letter “B” is incorrect because the mortgagor as the owner of the property mortgaged need not get the consent of the
creditor to sell his property. He enjoys all the privileges of an owner including the right to dispose.
Letter “C’ is incorrect because the agreement not to sell is null and void.
100
. Letter “C’ is the correct answer.
This is a reciprocal obligation where each of the parties is a promisee of a prestation and promises another in return as
a counterpart or equivalent of the other. The most salient of this obligation is reciprocity. The obligation of one party is
dependent on the obligation of another such that both obligations are to be performed simultaneously so that the
performance of one is conditioned upon the simultaneous performance of the other.
Although there is no fix date when there is a performance of the obligation of the seller and buyer begins, the obligation
of the seller to deliver the pick-up arises if the buyer is ready to perform the obligation that is to pay P250, 000.00.
Letter “A” is incorrect because, he power to rescind obligation is implied in reciprocal obligations, but only in case one of
the obligors should not comply with what is incumbent upon him (Article 1191, New Civil code).
Letter “B” is incorrect because the buyer need not write demand letter to the seller but there must rather be
simultaneous performance of the obligations.
Letter “D” is incorrect because unless there is performance on the part of one of the contracting parties. The other party
has no obligation yet, not withstanding if there is a meeting of minds between them.
101
. Letter “A” is the correct answer.
One of the requisite of a valid contract is that the consent, which constitutes an offer, must be certain and the
acceptance is absolute. A qualified acceptance constitutes a counter-offer (Article 1319 of the Civil Code). Here, by the
terms of the letter, there is only an offer to sell of piece of land for P500, 000.00. By increasing the amount to P700,
000.00 there is a qualified acceptance that he agrees to sell the lot but in a higher price, there is no meeting of the minds
yet. Thus, Mr. Bascon cannot compel Mr. Araneta to accept the P500, 000.00 and making him sign and execute a Deed
of Sale.
Letter “B’ is incorrect because nothing in a given problem which shows that Bascon accepted the offer of Araneta,
regardless of the period of two months.
Letter “C” is incorrect because Araneta is not estopped because his second letter constitute qualified acceptance and
not absolute.
Letter “D’ is incorrect because in absence of consent, there is no perfected contract.
102
. Letter “B” is the correct answer.
By express provision, a stipulation forbidding the owner from alienating the immovable mortgaged shall be void (Article
2130, New Civil Code). A property subject of mortgaged is never disposed of neither is there any transfer of ownership,
thus the mortgagor remains the owner and enjoys the rights and privileges afforded to an owner.
Letter “A” is incorrect because by specific provision of the law the stipulation forbidding the owner from alienating the
immovable mortgaged is void (ibid).
Letter “C” is incorrect because the mortgagor as the owner of the preppy mortgaged need not get the consent of the
creditor to sell his property. He enjoys all the privileges of an owner including the right to dispose.
Letter “D” is incorrect because Arco can sell the land even if he has not yet pay the obligation, any agreement to the
contrary is null and void.
103
. Letter “A” is the correct answer.
By a chattel mortgage, personal property is recorded in the Chattel Mortgager Registers as a security for the
performance of an obligation. If the movable is delivered to the creditor or a third person, the contract is pledge and not
chattel mortgage (Article 2140, New Civil Code).
Letter “B” is incorrect because in chattel mortgage the delivery of personal property is not necessary as in pledge, but
the thing must be recorded in the Chattel Mortgage Registry as provided.
Letter “C’ is incorrect because if the object of pledged is sold and the price of the sale is more than said amount, the
debtor shall not be entitled to the excess, unless it is otherwise agreed. If the price of the sale is less, neither shall the
creditor be entitled to recover the deficiency, notwithstanding any stipulation to the contrary (Article 2115, supra).
Letter “D” is incorrect because the sale of the thing pledged shall extinguish the principal obligation, whether or not the
proceeds of the sale are equal to the amount of the principal obligation, interest and expenses in a proper case (supra).
104

. Letter “C” is the correct answer.


If the object of pledged is sold and the price of the sale is more than said amount, the debtor shall not be entitled to the
excess, unless it is otherwise agreed. If the price of the sale is less, neither shall the creditor be entitled to recover the
deficiency, notwithstanding any stipulation to the contrary (Article 2115, New Civil Code).
By express provision of the law, both statements are correct.
105
. Letter “D” is the correct answer.
The common requisite for Pledge, Chattel Mortgage and Antichresis is that they are constituted to secure the fulfillment
of a principal obligation.
Letter “A” is incorrect because, in pledge, the thing pledge must be placed in the possession of the creditor or third party,
by common agreement, not just in writing (Article 2193, new Civil Code).
Letter “B” is incorrect because in mortgage the property must be recorded in the registry of the property not only that the
property must be in possession of the creditor to affect third person.
Letter “C” is incorrect because this is true only with mortgage but not width pledged and antichresis.
106
. Letter “C” is the correct answer.
It is the essence of the contract of pledge and mortgage that when the principal obligation becomes due, the thing in
which the pledge or mortgage consists may be alienated for the payment to the creditor (article 087, New Civil code).
Thus, it is true and correct that the pledgee and mortgagee may dispose the collateral when the principal obligation is
not paid when due, even if there is no agreement to that effect but by the operation of the law.
Letter “A” is incorrect because, the sale of the thing pledge shall extinguish the principal obligation, whether or not the
proceeds of the sale are equal to the amount of the principal obligation, and regardless of the agreement of the parties.
Letter “B” is incorrect because pledge and mortgage are constituted to secure the fulfillment of the obligation.
Letter “D” is incorrect because if the price of the sale is more than the principal obligation, the debtor shall not be entitled
to the excess, unless it is otherwise agreed upon (Article 2115, supra).
107

. Letter “D” is the correct answer.


That which is undertaken in fraud of creditors is rescissible contract and not void ab initio (Article 1381, New Civil Code).
Letters A, B, and C are contracts void ab initio, they are inexistent and void from the beginning due to lack of a valid and
legal objects, which is a requisite of a valid contract.
108
. Letter “D’ is the correct answer.
Unenforceable contracts are those that do not produce effect yet unless they are ratified.
Letter “A” is incorrect because, voidable contract are annullable contract, even though there may have been no damage
to the contracting parties (Article 1390, new Civil Code).
Letter “B” is incorrect because rescissible contracts are those validly agreed upon but may be rescinded or sat aside
under case provided by law (Article 1380, supra).
Letter “C” is incorrect because void contract cannot be ratified, neither can the right to set up the defense of illegality be
waived (Article 1409, supra).
109
. Letter “D” is the correct answer.
An oral agreement to answer all expenses for the wedding reception if A marries B is a valid contract though
unenforceable under the Statue of Fraud. This is an agreement made in consideration of marriage, other than a mutual
promise to marry (Article 1403 [c], New Civil Code).
Letter “A” is incorrect because when a sale of a piece of land or any interest therein is through an agent, the authority of
the latter shall be in writing, otherwise the sale shall be void (Article 1874, supra).
Letter “B” is incorrect because a contract of partnership is void whenever an immovable property is contributed thereto, if
an inventory of said property is not made, signed by the parties, and attached to the public instrument (Article 1773,
supra).
Letter “C” is incorrect for the same reason given in letter A.
110
. Letter “C” is the correct answer.
The remission of the whole obligation, obtained by one of the solidary debtors, does not entitle him to reimbursement
from his co-debtors (Article 1220, New Civil Code).
Letter “A” is incorrect because, A has no right to collect from B and C because he has not paid anything out of the
obligation but the same was extinguished by remission.
Letter “B” is incorrect because since there is entire remission of debt the obligation is extinguished regardless of whether
X pays Y of his share in the credit.
Letter “D” is incorrect for the same reason given in letter A.
111
. Letter “D” is the correct answer.
When the debtor binds himself to pay when his means permit him to do so, the obligation shall be deemed to be one
with a period (Article 1180, New Civil Code).
Letter “A” is incorrect because, in conditional obligation, the acquisition of rights, as well as the extinguishments or loss
of those already acquired, shall depend upon the happening of the event which constitutes the condition (Article 1181,
supra).
Letters B and c are incorrect because every obligation, whose performance does not depend upon the future or
uncertain event, or upon a past event unknown to the parties, is demandable at once (Article 1179, supra).
112
. Letter “C” is the correct answer.
Whoever voluntarily takes charge of the agency or management of the business or property of another, without any
power from the latter constitute negotiorum gestio.
Letter “A” is incorrect because, whoever by act or omission causes damage to another, there being fault or negligence is
oblige to pay for the damage done. Such fault or negligence, if there is no pre-existing contractual relation between the
parties, give rise to quasi delict.
Letter “B” is incorrect because a certain lawful, voluntary and unilateral acts give rise to the juridical relation of quasi
contract to the end that no one shall unjustly enriched or benefited at the expense of another (Article 2142, supra).
Letter “D” is incorrect because there is solutio indebiti if something is receive when there is no right to demand it, and it
was unduly delivered through mistake the obligation to return arises (Article 2154, supra).
113
. Letter “D” is the correct answer.

114
. Letter “B” is the correct answer.

115
. Letter “B” is the correct answer.

116
. Letter “C” is the correct answer.

117
. Letter “B” is the correct answer.

118
. Letter “B” is the correct answer.

119
. Letter “B” is the correct answer.

120
. Letter “A” is the correct answer.

121
. Letter “B” is the correct answer.

122
. Letter “C” is the correct answer.

123
. Letter “D” is the correct answer.

124
. Letter “A” is the correct answer.

125
. Letter “B” is the correct answer.

126
. Letter “C” is the correct answer.

127
. Letter “A” is the correct answer.
128
. Letter “D” is the correct answer.

129
. Letter “C” is the correct answer.

130
. Letter “B” is the correct answer.

131
. Letter “A” is the correct answer.

132
. Letter “C” is the correct answer.

133
. Letter “C” is the correct answer.

134
. Letter “B” is the correct answer.

135
. Letter “D” is the correct answer.

136
. Letter “A” is the correct answer.

137
. Letter “B” is the correct answer.

138
. Letter “D” is the correct answer.

139
. Letter “B” is the correct answer.

140
. Letter “B” is the correct answer.

141
. Letter “D” is the correct answer.

142
. Letter “B” is the correct answer.

143
. Letter “A” is the correct answer.

144
. Letter “B” is the correct answer.

145
. Letter “D” is the correct answer.
146
. Letter “C” is the correct answer.

147
. Letter “A” is the correct answer.

148
. Letter “A” is the correct answer.

149
. Letter “D” is the correct answer.

150
. Letter “C” is the correct answer.

151
. Letter “A” is the correct answer.

152
. Letter “B” is the correct answer.

153
. Letter “C” is the correct answer.

154
. Letter “A” is the correct answer.

155
. Letter “D” is the correct answer.

156
. Letter “D” is the correct answer.

157
. Letter “C” is the correct answer.

158
. Letter “D” is the correct answer.

159
. Letter “B” is the correct answer.

160
. Letter “C” is the correct answer.

161
. Letter “C” is the correct answer.

162
. Letter “C” is the correct answer.
163
. Letter “A” is the correct answer.

164
. Letter “A” is the correct answer.

165
. Letter “D” is the correct answer.

166
. Letter “A” is the correct answer.

167
. Letter “C” is the correct answer.

168
. Letter “D” is the correct answer.

169
. Letter “B” is the correct answer.

170
. Letter “D” is the correct answer.

171
. Letter “C” is the correct answer.

172
. Letter “C” is the correct answer.

173
. Letter “A” is the correct answer.

174
. Letter “A” is the correct answer.

175
. Letter “A” is the correct answer.

176
. Letter “C” is the correct answer.

177
. Letter “D” is the correct answer.

178
. Letter “C” is the correct answer.

179
. Letter “C” is the correct answer.

180
. Letter “A” is the correct answer.
181
. Letter “C” is the correct answer.

182
. Letter “D” is the correct answer.

183
. Letter “D” is the correct answer.

184
. Letter “B” is the correct answer.

185
. Letter “C” is the correct answer.

186
. Letter “B” is the correct answer.

187
. Letter “B” is the correct answer.

188
. Letter “B” is the correct answer.

189
. Letter “D” is the correct answer.

190
. Letter “D” is the correct answer.

191
. Letter “D” is the correct answer.

192
. Letter “A” is the correct answer.

193
. Letter “D” is the correct answer.

194
. Letter “D” is the correct answer.

195
. Letter “A” is the correct answer.

196
. Letter “A” is the correct answer.

197
. Letter “B” is the correct answer.

198
. Letter “A” is the correct answer.
199
. Letter “B” is the correct answer.

200
. Letter “B” is the correct answer.

201
. Letter “A” is the correct answer.

202
. Letter “B” is the correct answer.

203
. Letter “B” is the correct answer.

204
. Letter “D” is the correct answer.

205
. Letter “C” is the correct answer.

206
. Letter “C” is the correct answer.

207
. Letter “A” is the correct answer.

208
. Letter “B” is the correct answer.

209
. Letter “C” is the correct answer.

210
. Letter “B” is the correct answer.

211
. Letter “A” is the correct answer.

212
. Letter “D” is the correct answer.

213
. Letter “B” is the correct answer.

214
. Letter “A” is the correct answer.

215
. Letter “C” is the correct answer.
216
. Letter “C” is the correct answer.

217
. Letter “B” is the correct answer.

218
. Letter “D” is the correct answer.

219
. Letter “B” is the correct answer.

220
. Letter “C” is the correct answer.

221
. Letter “B” is the correct answer.

222
. Letter “D” is the correct answer.

223
. Letter “A” is the correct answer.

224
. Letter “B” is the correct answer.

225
. Letter “C” is the correct answer.

226
. Letter “D” is the correct answer.

227
. Letter “B” is the correct answer.

228
. Letter “D” is the correct answer.

229
. Letter “B” is the correct answer.

230
. Letter “A” is the correct answer.

231
. Letter “D” is the correct answer.

232
. Letter “C” is the correct answer.

233
. Letter “C” is the correct answer.
234
. Letter “C” is the correct answer.

235
. Letter “A” is the correct answer.

236
. Letter “B” is the correct answer.

237
. Letter “A” is the correct answer.

238
. Letter “C” is the correct answer.

239
. Letter “C” is the correct answer.

240
. Letter “B” is the correct answer.

241
. Letter “B” is the correct answer.

242
. Letter “B” is the correct answer.

243
. Letter “B” is the correct answer.

244
. Letter “D” is the correct answer.

245
. Letter “D” is the correct answer.

246
. Letter “C” is the correct answer.

247
. Letter “B” is the correct answer.

248
. Letter “C” is the correct answer.

249
. Letter “C” is the correct answer.

250
. Letter “C” is the correct answer.

251
. Letter “B” is the correct answer.
252
. Letter “A” is the correct answer.

253
. Letter “A” is the correct answer.

254
. Letter “A” is the correct answer.

255
. Letter “B” is the correct answer.

256
. Letter “B” is the correct answer.

257
. Letter “D” is the correct answer.

258
. Letter “B” is the correct answer.

259
. Letter “B” is the correct answer.

260
. Letter “B” is the correct answer.

Das könnte Ihnen auch gefallen